You are on page 1of 119

(https://www.project-management-prepcast.

com)

Here are Your Results


The summary is on the right and the details are below.

How will you know if you are ready to take "the real" PMP Exam? As a rule of thumb, we tell our students, that if you are consistently
scoring 75% or more with the PMP Exam Simulator when you take a simulated exam for the first time, then that is a good sign of
readiness. However, take this with a grain of salt because we are all individuals, and your personal readiness may differ.

Warning: The results you see on this page are temporary! If you restart your browser, or reload, close or move away from this page
in any way, then your results will disappear.

Please sign up for The PM Exam Simulator (/component/banners/click/16). It is a full-featured online testing environment for the PMP
Exam with over 2,100 questions where your results are stored permanently as well as sent to you via email.

Question 1 - Qid 6113151, Risk Management, 2. Process, 11.7 Monitor Risks

The project management plan specifies that a predictive development approach has been selected to produce the project deliverables.

Where in the project life cycle will the overall project risk be the lowest?

(Please note that on the real PMP exam you may be asked to provide your answer by clicking the correct area in the image. But here in the
simulator, we are asking you to select the answer below.)

A
B
C
D

Hint Marked

Your answer is correct


Explanation:

A life cycle of a typical traditionally managed project consists of initiation, planning, execution, monitoring and controlling, and
closure. Overall project risk is the effect of uncertainty on the project as a whole. Many various factors contribute to the overall
project risk: technical, managerial, commercial, external, to name but a few. Deliverables are not produced up until the project
enters its execution. Project objectives, such as scope, schedule, cost, quality, etc. are not being measured during project
initiation and planning. At that point, there are many unknowns, and it's even unclear if the project will meet its objectives.
Therefore, the overall project risk at the beginning of the project is the greatest. As the project progresses, decisions are made,
and deliverables and produced and accepted, the overall project risk decreases. During project closure, there are typically not too
many unknowns left (if at all). After the deliverables are accepted, and the project's objectives are met, the "victory" can be
declared. This is where the overall project risk is considered very low or non-existent. Therefore, of the choices provided, the risk
is the lowest during project closure (area D).

Please note that on this page we only show a general explanation for the 120 questions.

The PMP Exam Simulator (http://www.pm-exam-simulator.com/pmp), however, will give you a much richer experience
because you also receive an explanation of why each of the answer choices is correct or incorrect, as well as a reference that
allows you to read up on the subject and learn from any mistakes.

Question 2 - Qid 6110002, Cost Management, 3. Business Environment, 4.1 Develop Project Charter

A company is considering two projects, Alpha and Beta. Project Alpha is expected to result in a $50 million net profit, while project Beta and
is expected to net $45 million. Both projects could be very lucrative and rewarding. However, the financial controller has stated that the
company can only invest in one of these projects.

If project Alpha is selected, what will be the opportunity cost?

$95 million
$50 million
$45 million
$5 million

Hint Marked

Your answer is correct


Explanation:

Opportunity cost is regarded as the value of the alternative that is not chosen. If the decision is made to select project Alpha and
forego the $45 million in potential profit from project Beta, the opportunity cost of this decision is $45 million, the value of project
Beta.

The topic of this question is not covered in the PMBOK® Guide. However, the Project Management Professional (PMP)®
Examination Content Outline (ECO) indicates that while there are some commonalities between the PMBOK® Guide and the ECO,
the exam is not bound by the PMBOK® Guide. The list of enablers specified in the tasks of the ECO domains is not exhaustive
either. The ECO assumes that prospective PMP aspirants are familiar with other sources of information/preparation, including but
not limited to PMI Code of Ethics and Professional Conduct, PMI?s Practice Standards (e.g., Scheduling, Earned Value
Management, etc.); organizational behavior theories such as Tuckman?s Ladder, Theory X and Theory Y, Maslow's hierarchy of
needs; commonly frowned upon project management practices, such as gold plating; and others. We intentionally have these
questions in our simulator so that you would be better prepared for the real exam. PMP aspirants are encouraged to explore
additional sources of information and/or to be familiar with them from their project management experience.

Question 3 - Qid 6110003, Integration Management, 2. Process, 4.1 Develop Project Charter

You are managing a project to build five bridges. The project consists of five sequential phases with each phase delivering one bridge.

After the first phase of the project is complete, which process group of the second phase should follow?

Initiating
Planning
Executing
Closing

Hint Marked

Your answer is correct


Explanation:
The Initiating Process Group is made up of two processes, Develop Project Charter and Identify Stakeholders. The Initiating
Process Group is performed to define a new project or a new phase of an existing project by obtaining authorization to start the
project or phase. The purpose of this Process Group is to align the stakeholders' expectations and the project purpose, inform
stakeholders of the scope and objectives, and discuss how their participation in the project and its associated phases can help to
ensure their expectations are met.

In the scenario described, you are about to start a new project phase. Therefore, you should start with the Develop Project Charter
process. It does not necessarily mean you would develop a new project charter for each consecutive project phase, but it means
that you will perform the processes of the Initiating Process Group starting with the Develop Project Charter process. You will
review the project charter to see if the high-level assumptions and constraints made during project initiation are still valid, and
whether or not the market conditions are still favorable for the project. Performing this process may result in an update of the
charter or an addendum to the project charter. In an extreme scenario, this process may result in project termination.

While the PMBOK® Guide does not explicitly mention going back to the Initiating Process Group to look at the project charter, it
does provide several examples implying that the project team may choose to go with each new phase as a separate, stand-alone
project. Undertaking a stand-alone project means performing all its processes, starting with the Develop Project Charter. The
same logic applies to the next process in the Initiating Process Group, the Identify Stakeholders process. It is possible that new
stakeholders became involved in the project, or some of the previous stakeholders left, or the level of engagement of the existing
stakeholders have changed. Even if nothing has changed, the Develop Project Charter and the Identify Stakeholders processes
should be performed. Otherwise, how would you be able to conclude that nothing has changed? Therefore, starting the new
phase of the project with the processes of the Initiating Process Group is the best answer to the question asked.

Question 4 - Qid 6110005, Stakeholder Management, 2. Process, 13.1 Identify Stakeholders

The project aims to investigate the feasibility of constructing a hydroelectric dam from a technical, economic, and social point of view. The
project charter created by the project manager on behalf of the project sponsor is now approved.

Which of the following processes should be performed next?

Develop Project Management Plan


Identify Risks
Identify Stakeholders
Develop Project Charter

Hint Marked

Your answer is correct


Explanation:

The Identify Stakeholders process is the process of identifying all people or organizations impacted by the project and
documenting relevant information regarding their interests, involvement, and impact on project success. Both the Develop
Project Charter process and the Identify Stakeholders process are part of the Initiating Process Group which should be completed
before proceeding to the processes in the Planning Process Group. In this scenario, since the project charter has been developed
and approved, the Identify Stakeholders process should be performed next.

Question 5 - Qid 6110006, Stakeholder Management, 2. Process, 13.1 Identify Stakeholders

You are managing a software upgrade project for your company. Your project sponsor has a great deal of authority regarding project
decisions, but you recently discovered that he doesn't have much interest in your project. To complete the project successfully, you need
your project sponsor's continuous support.

What stakeholder management strategy should you use with your project sponsor?

Keep the sponsor satisfied


Manage the sponsor closely
Monitor the sponsor's actions
Keep the sponsor informed

Hint Marked

Your answer is correct


Explanation:

A power/interest grid is a classification model used for stakeholder analysis and management that groups the stakeholders based
on their level of authority (power) and their level of concern (interest) regarding project outcomes. The classification and
recommended management strategies are as follows:

Low-power/low-interest: monitor only


Low-power/high-interest: keep informed
High-power/low-interest: keep satisfied
High-power/high-interest: manage closely

Based on this scenario, the sponsor can be classified as high-power and low-interest; therefore, keeping the sponsor satisfied is
the best strategy under the circumstances described in the scenario.

Question 6 - Qid 6110008, Scope Management, 2. Process, 5.2 Collect Requirements

The project sponsor asks for a copy of the document that contains the description, owner, source, priority, and status of product
requirements.

Which project document is the project sponsor requesting?

The requirements management plan


The requirements traceability matrix
The scope management plan
The work breakdown structure (WBS)

Hint Marked

Your answer is incorrect


Explanation:

In this scenario, the project sponsor requested a document that provides specific information about the product requirements of
your current project. The requirements traceability matrix is a grid that links product requirements from their origin to the
deliverables that satisfy them. Typical attributes used in the requirements traceability matrix may include a unique identifier, a
textual description of the requirement, the rationale for inclusion, owner, source, priority, version, current status, and status date.
Additional attributes to ensure that the requirement has met stakeholders' satisfaction may include stability, complexity, and
acceptance criteria.

Question 7 - Qid 6110011, Schedule Management, 2. Process, 6.2 Define Activities

You are facing problems decomposing the testing work package into the final activities required to complete the work package. Detailed
testing plans and activities cannot be determined until the system is at least 50% developed and more details become available. The
system development work package will take at least three months to complete.

What is the best way to resolve this problem?

Use product analysis techniques such as product breakdown and systems analysis to decompose the system testing work
package
Decompose the system development work package now and decompose the system testing work package later
Break down the project into multiple phases so that the system testing work package goes into the second project phase
Consult the project management plan to determine what to do in this situation

Hint Unmarked

Your answer is incorrect


Explanation:

Delaying the decomposition of the system testing work package until later in the project is an example of rolling wave planning.
Rolling wave planning is an iterative planning technique in which the work to be accomplished in the near term is planned in
detail, while the work in the future is planned at a higher level. Decomposition may not be possible for a deliverable or
subcomponent that will be accomplished far into the future. The project management team usually waits until the deliverable or
subcomponent is agreed on, so the details of the work breakdown structure (WBS) can be developed. Therefore, the best
approach is to use rolling wave planning and decompose the system development work package now, and then decompose the
system testing work package later, when more project information becomes available.

Question 8 - Qid 6110014, Integration Management, 2. Process, 4.2 Develop Project Management Plan

You are about to start planning for a large and complex project. Due to the size of the project along with the regulatory and environmental
considerations, the development of a detailed project management plan will be critical.

As a starting point for initial project planning, what is the first thing you should do?

Conduct a project kick-off meeting to inform and engage stakeholders and gain commitment
Review the project charter to understand the high-level information about the project
Begin the process of identifying stakeholders so they can be engaged as necessary for project planning
Share the project scope statement with the project team for a common understanding of project deliverables

Hint Unmarked

Your answer is incorrect


Explanation:

The question states that project planning is about to begin which suggests that project initiation has been completed. The project
charter is created during the Develop Project Charter process as part of project initiation. The project team uses the project
charter as a starting point for initial project planning. The type and amount of information in the project charter varies depending
on the complexity of the project and the information known at the time of its creation. At a minimum, the project charter should
define the high-level information about the project that will be elaborated in the various components of the project management
plan.

Question 9 - Qid 6110020, Resource Management, 2. Process, 9.0 Project Resource Management

As you are monitoring the cost baseline for your project, you notice that project expenses are higher than expected at this point in the
project.

What is the most likely cause of the variance?

Overestimated material costs


Lack of stakeholder support
An incomplete milestone list
Excess inventory

Hint Unmarked

Your answer is incorrect


Explanation:

Physical resource management is concentrated in allocating and using the physical resources such as materials, equipment, and
supplies efficiently and effectively. Organizations should have data on how much inventory to keep on hand as the project
progresses. Failing to manage and control resources efficiently can be a source of increased costs in a project. In the scenario
described, you have determined expenses are higher than expected. One of the causes for increased costs is failing to maintain
appropriate levels of inventory. Too much stock on hand may result in realizing the expenses for those supplies before they were
expected when comparing against the cost baseline.

Question 10 - Qid 6110029, Resource Management, 1. People, 9.4 Develop Team

Recently, you promoted your star programmer, Sam, to lead the agile development team. Sam was a fantastic software coder, and you
thought the promotion would mean that he could now share his expertise with the other development team members. However, you are
surprised to learn that the performance of Sam in his new role is not meeting your expectations.

Which empirical rule did you forget while considering the promotion of Sam?
Expectancy theory
Halo effect
Pareto concept
Murphy's law

Hint Unmarked

Your answer is incorrect


Explanation:

The halo effect (also known as the Peter principle) states "In a hierarchy, every employee tends to rise to his/her level of
incompetence." Most project team members are motivated by an opportunity to grow, accomplish, and apply their professional
skills to meet new challenges. Their achievements continuously promote them within an organization to a certain level until they
are unable to perform. Sam might have risen to his incompetency level, which is what you have likely forgotten to consider while
promoting him to lead the development team.

The topic of this question is only briefly mentioned in the PMBOK® Guide. However, the Project Management Professional (PMP)®
Examination Content Outline (ECO) indicates that while there are some commonalities between the PMBOK® Guide and the ECO,
the exam is not bound by the PMBOK® Guide. The list of enablers specified in the tasks of the ECO domains is not exhaustive
either. The ECO assumes that prospective PMP aspirants are familiar with other sources of information/preparation, including but
not limited to PMI Code of Ethics and Professional Conduct, PMI?s Practice Standards (e.g., Scheduling, Earned Value
Management, etc.); organizational behavior theories such as Tuckman's Ladder, Theory X and Theory Y, Maslow's hierarchy of
needs; commonly frowned upon project management practices, such as gold plating; and others. We intentionally have these
questions in our simulator so that you would be better prepared for the real exam. PMP aspirants are encouraged to explore
additional sources of information and/or to be familiar with them from their project management experience. The PMBOK® Guide
is provided as a reference to indicate that the scenario refers the Develop Team process.

Question 11 - Qid 6110032, Communications Management, 2. Process, 10.3 Monitor Communications

You have just taken over a project in execution. During your first week, you determine that the project team members spend a lot of time
responding to unexpected requests for information from different project stakeholders. Because of these interruptions, the team cannot
focus on their project activities, and as a result, the project is behind schedule.

What should you do first?

Tell the team members to stop responding to unexpected requests since it is not the responsibility of the team
Push the team to respond to all the requests faster so that they can perform their assigned project activities on time
Ask the team to forward all stakeholder inquiries to you so that the team members can focus on their assigned tasks
Review the communications management plan to understand how stakeholder requests should be handled

Hint Unmarked

Your answer is incorrect


Explanation:

Monitor Communications is the process of monitoring and controlling communications throughout the entire project life cycle to
ensure the information needs of the project and its stakeholders are met. In this scenario, the stakeholders are reaching out
directly to the project team members, which is not the most effective use of the team's time. To determine why this is happening,
you need to review the communications management plan to determine how stakeholder inquiries are to be managed. With this
information, you can best judge whether the stakeholders are violating that agreement. After that, you can proceed to make
whatever changes are needed to shield the project team from the time-consuming responses to the stakeholders' requests for
information and develop a better communications protocol for stakeholder information requests.

Question 12 - Qid 6110034, Scope Management, 2. Process, 5.1 Plan Scope Management

A team member submits a draft of the requirements management plan to the project manager for a review. The project manager notices
that one of the components included in the plan does not belong there.

Which of the following components did the project manager most likely notice?
A process that describes how the project requirements will be prioritized
The metrics that will be used and the rationale for using them
A process that specifies how a project scope statement will be prepared
How requirement activities will be planned, tracked, and reported

Hint Unmarked

Your answer is incorrect


Explanation:

The requirements management plan is a component of the project or program management plan that describes how
requirements will be analyzed, documented, and managed. A process that specifies how a project scope statement will be
prepared is an element of the scope management plan and should be excluded from the requirements management plan. The
incorrect answer choices represent components that should be included in the requirements management plan.

Question 13 - Qid 6110038, Scope Management, 2. Process, 4.5 Monitor and Control Project Work

While reviewing your project's performance, you discover a significant variance. If the issue is not fixed before the next customer inspection,
your project might be canceled. However, after a further review conducted by the project team, you are glad to hear that bringing the
project back on track requires only a minor correction to a configuration element.

What should you do first?

Implement the corrective action


Submit a change request
Consult the project sponsor
Delay the customer inspection

Hint Unmarked

Your answer is incorrect


Explanation:

All of the available answers seem like possible options in this scenario, but the question is specifically asking what you should do
'first'. Although you have already analyzed the situation and you know what corrective actions are required, you must follow the
proper change control procedure. A configuration element is a project artifact that has been placed under configuration control.
Any change to a configuration element should be formally controlled and will require a change request. Since the question
describes a situation requiring a change to a configuration element, the first thing you should do is submit a change request and
have it approved before you can implement the changes.

Question 14 - Qid 6110039, Integration Management, 2. Process, 4.5 Monitor and Control Project Work

A project manager is leading a process improvement project for a factory operation. Currently, the project manager and the team are
performing the Monitor and Control Project Work process.

Which of the following activities might be carried out during this process?

Comparing actual project performance against the project management plan


Implementing approved change requests to achieve the project's objectives
Analyzing change requests and either approving or rejecting them
Gaining formal acceptance of the deliverables by the customer or sponsor

Hint Unmarked

Your answer is incorrect


Explanation:
Monitor and Control Project Work is the process of tracking, reviewing, and reporting overall progress to meet the performance
objectives defined in the project management plan. Of the available choices, only comparing actual project performance against
the project management plan is performed during the Monitor and Control Project Work process.

Question 15 - Qid 6110040, Integration Management, 2. Process, 4.6 Perform Integrated Change Control

An organization has a policy that a change control board (CCB) must review all change requests for every project in the organization, and
that the CCB follows the recommended activities outlined in the PMBOK® Guide. Project managers and the project team do not serve as
members of the CCB.

Based on this scenario, which of the following activities might be performed by members of the CCB?

Submitting a change request


Implementing approved change requests
Initiating and closing a project
Monitoring implementation of approved changes

Hint Unmarked

Your answer is incorrect


Explanation:

The change control board (CCB) is a formally chartered group responsible for reviewing, evaluating, approving, deferring, or
rejecting change requests. The CCB can include project stakeholders, and a change may be requested by any stakeholder involved
with the project. Therefore, it is possible for a member of the CCB to submit a change request.

Question 16 - Qid 6110041, Integration Management, 2. Process, 4.6 Perform Integrated Change Control

A company has authorized a dedicated change control board (CCB) to review and evaluate all change requests that are submitted for any
project.

If the project sponsor is a member of the CCB, which of the following best describes an individual or group that can request a change on a
project?

Project manager
Project sponsor
Project team
Any stakeholder

Hint Unmarked

Your answer is incorrect


Explanation:

Changes may be requested by any stakeholder involved with the project and may occur at any time throughout the project life
cycle. The project manager, the project sponsor, and the project team are all project stakeholders and therefore, can request
changes. All of the answer choices are technically correct, but the option of 'any stakeholder' provides a complete answer and is,
therefore, the best answer to the question. Additionally, the fact that the company authorizes a change control board (CCB) and
the project sponsor is its member is irrelevant to the question asked. It is essential to discern the relevant information of a
question to select the best answer of the choices given.

Question 17 - Qid 6110055, Procurement Management, 2. Process, 12.3 Control Procurements

One of your contractors is not performing to the agreed-upon standards. You want to terminate the contract and engage another contractor
for the same job. You need to determine the specific legal procedure for the early termination of the contract.

What should you do?

Check the procurement management plan


Look at the scope statement
Review the risk register
Consult the contract

Hint Unmarked

Your answer is incorrect


Explanation:

The contract is the document you would refer to in order to understand the early termination procedure. A contract is a mutually
binding agreement that obligates the seller to provide the specified deliverable. It constitutes a legal relationship. The early
termination of a contract is a special case of the Control Procurements process that can result from an agreement by both parties,
from the default of one party, or for the convenience of the buyer if provided for in the contract. The rights and responsibilities of
the parties in the event of an early termination are contained in the termination clause of the contract. 

Question 18 - Qid 6110058, Stakeholder Management, 2. Process, 13.1 Identify Stakeholders

A project is divided into four sequential phases. The second phase of the project has just been completed, and the third phase is about to
begin.

What should the project manager do first?

Begin the process of directing and managing the project work


Perform the process of identifying the project stakeholders
Start the process of monitoring and controlling the project work
Begin the process of developing the project management plan

Hint Unmarked

Your answer is incorrect


Explanation:

The process of identifying project stakeholders occurs for the first time in a project either before or at the same time the project
charter is developed and approved. It is repeated as necessary but should be performed at the start of each phase and when a
significant change in the project or organization occurs. Each time the identification process is repeated, the project management
plan components and project documents should be consulted to identify relevant project stakeholders.

Question 19 - Qid 6110061, Stakeholder Management, 2. Process, 13.1 Identify Stakeholders

An organization has a project management office (PMO) that ensures all relevant project documents are created, and all project
management processes are followed according to established best practices.

What should the project manager review first to identify stakeholders for a newly started project?

The business documents as a source of information about the project's stakeholders


The communications management plan to learn about the project's stakeholders
The stakeholder engagement plan to identify the management strategies and actions required to engage stakeholders
effectively
The requirements documentation for information on potential stakeholders

Hint Unmarked

Your answer is incorrect


Explanation:

The business documents include the business case and the benefits management plan. Both of these documents are created
during pre-project work and are available as sources of information about the project's stakeholders during the first iteration of
the Identify Stakeholders process. The business case identifies an initial list of stakeholders affected by the project. The benefits
management plan may identify the individuals and groups that will benefit from the delivery of the outcomes of the project and
are thus considered as stakeholders. Among the available options, only the business documents are available as sources of
relevant information during the first iteration of the Identify Stakeholders process.

Question 20 - Qid 6112348, Schedule Management, 2. Process, 6.5 Develop Schedule

The following table is an activity list that includes the duration and predecessor for a series of project activities.

What is the duration of the critical path?

11
14
9
13

Hint Unmarked

Your answer is incorrect


Explanation:

Using the critical path method may require drawing a project schedule network diagram, or listing all the possible the sequences
of paths with the given activities, then adding up the durations, and knowing which path constitutes the critical path.

Given the activity list shown, you can see that:

Start serves as predecessor for both A and E, creating a divergence into two paths. 
Activity A serves as predecessor for both B and C, creating another divergence.
Activity E serves as a predecessor for both B and F, creating another divergence.

The project schedule network diagram is as follows.


The resulting list of paths with their aggregated durations is below. The critical path is Start-E-B-D-End, and the duration of the
critical path with the data given is 14.

Question 21 - Qid 6110065, Stakeholder Management, 2. Process, 13.1 Identify Stakeholders

A project manager is in the process of developing the stakeholder register. As the first step in this process, she needs to create a list of
potential stakeholders and decides to use a data gathering technique called brain writing. 

What is the first thing to do when using that technique?

Convene a data gathering meeting with the participants


Develop your own template for a stakeholder register
Send a list of questions to participants before the creativity session
Consult the stakeholder engagement plan to understand how the stakeholders are to be identified

Hint Unmarked

Your answer is incorrect


Explanation:

Data gathering techniques that can be used for the Identify Stakeholders process include questionnaires and surveys,
brainstorming, and brain writing. Brain writing is a refinement of brainstorming that allows individual participants time to
consider the questions individually before the group creativity session is held. Therefore, the first step of brain writing is providing
the participants with the questions before the creativity session. This process will allow the participants to be better prepared
which will make the meeting more effective than a traditional brainstorming session.

Question 22 - Qid 6110066, Communications Management, 2. Process, 10.0 Project Communications Management

A project manager does most of her communicating with team members, internal project stakeholders, and external project stakeholders
via email. The project manager incorporates specific techniques when composing her emails to reduce any misunderstandings.

Which of the following is not considered one of these techniques?

Concise expression
Clear purpose directed to the needs of the reader
Controlled flow of words and ideas
Communications register

Hint Unmarked

Your answer is incorrect


Explanation:

The scenario describes the use of techniques that will aid in the reduction of misunderstandings in written communication
specifically. These techniques are known as the 5Cs of written communication:

1. Correct grammar and spelling


2. Concise expression and elimination of excess words
3. Clear purpose and expression directed to the needs of the reader
4. Coherent, logical flow of ideas
5. Controlled flow of words and ideas
However, the question asks which answer is 'not' a technique that will aid in the reduction of misunderstandings in written
communication, or in other words, which answer is not one of the 5Cs of written communication. 'Communications register' is not
one of the 5Cs components. Actually, 'communications register' is a made-up term.

Note, the term 'tools and techniques' used in this question to describe the 5Cs does not refer to the traditional meaning of tools
and techniques of the project management processes. We are using this term as we consider it to be a reasonable description of
the components comprising the 5Cs.

Question 23 - Qid 6110070, Risk Management, 2. Process, 11.3 Perform Qualitative Risk Analysis

As the project manager, you are leading a brainstorming session with key stakeholders to identify project risks. Once the project risks are
determined, you immediately proceed to nominate different team members as risk owners and assign them the task of developing
individual risk response plans.

Have you properly conducted risk management planning?

No, you should be the one developing the risk response plans as you are ultimately responsible for the outcomes of the
project.
Yes, the risk response plans are an output of the Identify Risks process and should be developed right after the risks have
been identified.
No, before assigning risk responsibility to team members, you need to assess each risk's probability of occurrence and its
impact.
Yes, the team members should be assigned responsibility for a risk as soon as possible and tasked with creating a risk
response plan.

Hint Unmarked

Your answer is incorrect


Explanation:

To answer this question correctly, you need to understand what process the project team is currently performing. In this scenario,
you have identified project risks and therefore have finished creating the initial version of the risk register. This conclusion means
that you have completed the Identify Risks process, which is one of the planning processes in the Project Risk Management
Knowledge Area. According to the PMBOK® Guide, project risk management processes occur in the following order: 1. Plan Risk
Management 2. Identify Risks 3. Perform Qualitative Risk Analysis 4. Perform Quantitative Risk Analysis 5. Plan Risk Responses 6.
Implement Risk Responses 7. Monitor Risks. Your next step is, therefore, to move into the Perform Qualitative Risk Analysis
process, which is the process where you will prioritize risks for further analysis or action by assessing and combining their
probability of occurrence and impact. The Perform Qualitative Risk Analysis process and, if needed, the Perform Quantitative Risk
Analysis process should be done after the Identify Risks process in order to properly identify the risks on which to focus and plan.
Only then should team members be assigned risk ownership. Assigning risk ownership to team members before determining
which risks are the most likely to occur and impact the project is not an efficient use of resources and is not the proper protocol
outlined in the PMBOK® Guide.

Question 24 - Qid 6110072, Scope Management, 2. Process, 5.2 Collect Requirements

You are managing a project to build a database and need to acquire information from the stakeholders of the project. The team is using
brainstorming, focus groups, interviews, questionnaires, and benchmarking to understand and document the stakeholders' needs.

The methods the team is using are representative of which tool and technique?

Data gathering
Data analysis
Decision making
Data representation

Hint Unmarked

Your answer is incorrect


Explanation:
The project described in the scenario is in the Collect Requirements process where project stakeholders' needs and requirements
are determined and documented. A tool and technique used in the Collect Requirements process to elicit requirements from
stakeholders is data gathering. Data gathering includes brainstorming, focus groups, interviews, questionnaires, surveys, and
benchmarking.

Question 25 - Qid 6110074, Scope Management, 2. Process, 5.4 Create WBS

The project is in execution, but your project team is unable to stop arguing about the project milestones and the specific risks associated
with them. This on-going conflict puts the project at risk of running behind schedule.

What would likely have helped avoid this conflict had it been properly created first?

A project charter
Physical resource assignments
Quality metrics
A work breakdown structure

Hint Unmarked

Your answer is incorrect


Explanation:

The WBS is considered by many project managers to be the heart of the projects lead by a predictive project management
approach since a WBS lists the total scope of work to be carried out by the project team to accomplish the project objectives and
create required deliverables. The milestone list is an output from the Define Activities process to which the WBS serves as an input
(as part of the scope baseline). The risk register is an output from the Identify Risks process to which the WBS serves as an input
(as part of the scope baseline) used as a framework to structure risk identification techniques. Had the project manager and the
team taken the time to create a proper WBS, it is likely the team would have had a more accurate milestone list and risk register
with which to finish their project management plan on time and avoid the conflict described in the scenario.

Question 26 - Qid 6110075, Scope Management, 2. Process, 4.5 Monitor and Control Project Work

During project execution, the team discovers that some of the buttons on the remote control developed by the project for a new product do
not work according to the specifications. A change request is submitted and approved to replace the buttons.

Which of the following best describes this change request?

Corrective action
Preventive action
Defect repair
Gold plating

Hint Unmarked

Your answer is incorrect


Explanation:

Approved change requests can be a corrective action, a preventive action, or a defect repair. The question reads, "some of the
buttons on the remote control developed by the project for a new product do not work according to the specifications". This
scenario is an example of a defect repair since it involves taking immediate action to correct a defective product component that
does not meet product specifications. The performing organization is neither taking preventive action nor addressing a deviation
(corrective action) from the agreed-upon project baselines. Gold plating does not apply here since the organization is adding extra
buttons to the remote control unit not as a means of enhancing the product or making the customer happy, but as a means of
meeting product specifications.
Question 27 - Qid 6110076, Schedule Management, 2. Process, 6.4 Estimate Activity Durations

A project manager meets with a customer for initial discussions about an upcoming project. At the end of the meeting, the customer asks
the project manager for a rough estimate of the project duration. Based on her experience with three similar projects, the project manager
provides an estimate of 8-10 months.

What estimation technique does the project manager use?

Expert judgment
Three-point estimating
Analogous estimating
Bottom-up estimating

Hint Unmarked

Your answer is incorrect


Explanation:

The project manager is using historical duration information from comparable projects to estimate the duration of a future
project. This gross value estimating approach is called analogous estimating and is frequently used to provide a ballpark figure
when there is a limited amount of detailed information available about the project.

Question 28 - Qid 6110079, Quality Management, 2. Process, 8.2 Manage Quality

A project manager is performing quality assurance for a project whose quality standards are very stringent. In order to meet and exceed the
quality requirements, the project manager uses a set of technical guidelines, which were applied during the design phase of the project to
optimize quality.

What tool or technique is the project manager using in this scenario?

Design for X (DfX) where the X in DfX represents the quality aspect of the product
Design of experiments (DOE) as a quality improvement methodology
The Manage Quality process to ensure compliance with project quality requirements
Process analysis to identify opportunities for process improvements

Hint Unmarked

Your answer is incorrect


Explanation:

The question implies that the Manage Quality process, sometimes referred to as quality assurance, is underway. Design for X (DfX)
is one of the tools and techniques which may be used during the Manage Quality process. DfX is a set of technical guidelines that
may be applied during the design of a product for the optimization of a specific aspect of the design. DfX can control or even
improve the product's final characteristics. The X in DfX can be different aspects of product development which can include but
are not limited to quality improvement.

Question 29 - Qid 6110081, Stakeholder Management, 2. Process, 13.1 Identify Stakeholders

You have compiled a list of potential stakeholders for your project, and now you are classifying the stakeholders with regard to the direction
of their influence. As you work through your way down the list of stakeholders, you have come to the project sponsor.

How should this stakeholder be classified?

Downward
Sideward
Outward
Upward

Hint Unmarked
Your answer is incorrect
Explanation:

According to the scenario, you are carrying out the Identify Stakeholders process. Directions of influence is an example of a data
representation technique that can be used as part of this process. This technique classifies stakeholders according to their
influence on the work of the project or the project team itself. Stakeholders can be classified as upward, downward, outward, or
sideward. The upward classification includes senior management of the performing organization or customer organization,
sponsor, and steering committee. Therefore, the project sponsor should be classified as upward.

Question 30 - Qid 6110083, Scope Management, 2. Process, 4.5 Monitor and Control Project Work

During project planning, the project team rigorously defined the project scope. During project execution, a senior engineer calls the project
manager and suggests a simpler and more efficient design which could benefit the project.

What should the project manager do next?

Nothing, no changes are needed since the project scope was rigorously defined
Discuss the change with the project sponsor
Evaluate the impact of the proposed change on the project constraints
Ask that the engineer submit a change request

Hint Unmarked

Your answer is incorrect


Explanation:

A change request is a document to record an adjustment to project scope, schedule, cost, etc. Changes may include corrective
action, preventive action, or defect repair, among others. The engineer is proposing what she believes to be a valid design change
which can be further evaluated by the change control board. Although changes may be initiated verbally, they should be recorded
in written form and entered into the change management and/or configuration management system. Thus, of the available
choices, asking the engineer to create and submit a change request is what the project manager should do next, and is, therefore,
the best answer to the question asked.

One may argue that evaluating the change should come before submitting the change request. We had an extensive debate on
the subject of change requests in general and on this question in particular. The reason being is that the PMBOK® Guide does not
do a good job in explaining the exact process in detail. We have discussed this process among our team of certified project
management professionals and have concluded that the first thing that should be done when a change is requested on a project
is the physical creation of the change request, a document if you wish, filling out a form. Otherwise, on what basis would a project
manager and the project team spend time evaluating a request that is not even documented?

Question 31 - Qid 6110087, Schedule Management, 2. Process, 6.4 Estimate Activity Durations

You are working with a team to estimate how much time is required for each activity. You are using a document that contains information
on the categories of labor, material, and equipment required to complete the project. You are also interested in the skill levels, required
certifications, and the grade levels for supplies.

Which of the following documents would you primarily refer to for this purpose?

Resource breakdown structure


Organizational breakdown structure
Work breakdown structure
Risk breakdown structure

Hint Unmarked

Your answer is incorrect


Explanation:
Based on the scenario described, the project team is performing the Estimate Activity Durations process. An input to this process
is the resource breakdown structure (RBS), which is a hierarchical list of team and physical resources related by category and
resource type that is used for planning, managing, and controlling project work. Each descending level represents an increasingly
detailed description of the resource until the information is small enough to be used in conjunction with the WBS to allow the
work to be planned, monitored, and controlled.

Question 32 - Qid 6110090, Risk Management, 2. Process, 11.2 Identify Risks

While performing project work, a team member notices that there is a potential new risk, which could impact the project's critical path.

What should the team member do immediately?

Recommend the project be placed on hold for a day to avoid the risk
Document the risk in the risk register
Draft an emergency fallback plan since the critical path is in jeopardy
Generate a risk report using quantitative and quantitative analysis of the identified risk to alert the team of the schedule
impacts

Hint Unmarked

Your answer is incorrect


Explanation:

By recording identified risks in the risk register, the project manager and team are able to keep track of all potential threats and
opportunities. Furthermore, once a risk has been analyzed using qualitative and or quantitative risk analysis methods, the risk
register serves as the central repository for planned risk responses and lists risk owners. As such, any time a new risk emerges on a
project, the first step to take is to immediately document it in the risk register.

Question 33 - Qid 6112005, Procurement Management, 2. Process, 12.3 Control Procurements

You are managing a project that is nearing its end and planning to settle all pending claims with the vendors. In one case, you are not
satisfied with the delivery of their final product. All conversations with the vendor have ended in an impasse.

What should you do in this situation?

Pay the vendor as some products were delivered


Take the vendor to court
Discontinue further meetings with the vendor and hold back payment
Identify some form of alternative dispute resolution

Hint Unmarked

Your answer is incorrect


Explanation:

Contested charges where the buyer and seller cannot reach an agreement are called claims. The contract usually has terms to
handle claims administration. If the parties cannot resolve the claim, it should be handled by alternative dispute resolution (ADR),
typically following procedures established in the contract. There are two types of ADR, arbitration and mediation. Direct
negotiation between the parties is the preferred method. Litigation in the courts is the last option that a project manager should
choose. Based on the scenario described, the project manager has negotiated directly with the seller. However, all conversations
have ended in an impasse. In this situation, the next step for the project manager to take is to identify some form of alternative
dispute resolution.

Question 34 - Qid 6112009, Risk Management, 2. Process, 11.0 Project Risk Management

Your company was awarded a contract that includes a 10% bonus payment if the project is completed two months early. You consider
several options to exploit this opportunity.

What is the best course of action for you to increase the chances of receiving the bonus?
Reduce project scope
Level resources
Crash the schedule
Purchase insurance

Hint Unmarked

Your answer is incorrect


Explanation:

This question involves two project management Knowledge Areas, Risk Management and Schedule Management. Crashing is an
example of a schedule compression technique that can be used during the Develop Schedule process. This technique may help
exploit the opportunity by adding more resources to the critical path. The processes of the Project Risk Management Knowledge
Area aim to develop options and strategies to capture opportunities (positive risks) and minimize the effects of threats (or
negative risks). In the scenario described, you have been presented with a chance to collect a bonus payment for early project
completion. A contingency reserve including amounts of time, money, or resources to take advantage of the opportunity should
be established. The opportunity to collect the bonus payment is within reach; therefore, the best course of action in this situation
is to crash the schedule to shorten the critical path.

Question 35 - Qid 6112010, Quality Management, 2. Process, 8.2 Manage Quality

The quality management plan specifies the use of a data representation technique to show the strength of relationships between four
factors.

Which of the following tools or techniques should be used to comply with the plan?

Affinity diagram
Matrix diagram
Scatter diagram
Quality diagram

Hint Unmarked

Your answer is incorrect


Explanation:

A matrix diagram seeks to show the strength of relationships among factors, causes, and objectives that exist between the rows
and columns that form the matrix. Depending on how many factors may be compared, the project manager can use different
shapes of matrix diagrams; for example, L, T, Y, X, C, and roof-shaped. An X-shaped matrix can display four factors for comparison
which is what required by the question, making the matrix diagram the best answer of those provided.

Question 36 - Qid 6112012, Stakeholder Management, 2. Process, 13.1 Identify Stakeholders

You have just taken over a project in which some of the deliverables will be produced by external vendors. You want to familiarize yourself
with those who are involved in the project from the sellers' side.

Where would you look to obtain this information?

Bid documents
Stakeholder register
Seller proposals
Stakeholder engagement plan

Hint Unmarked

Your answer is incorrect


Explanation:
The stakeholder register is a project document that contains relevant information regarding stakeholder interests, involvement,
interdependencies, influence, and potential impact on project success. Vendors are project stakeholders and as such should be
included in the stakeholder register. The project manager and project team members should refer to this document when
obtaining pertinent information about project stakeholders whether the stakeholders are internal or external to the performing
organization.

Question 37 - Qid 6112018, Stakeholder Management, 2. Process, 13.0 Project Stakeholder Management

Your project has been plagued with numerous change requests.

What is the most likely reason?

Incomplete stakeholder register


Lack of a detailed procedure to close the project
Failure to implement risk responses
Change management plan did not authorize a change control board

Hint Unmarked

Your answer is incorrect


Explanation:

Academic research and analyses of high-profile project disasters highlight the importance of a structured approach to the
identification, prioritization, and engagement of all stakeholders. The ability of the project manager and team to correctly identify
and engage all stakeholders in an appropriate way can mean the difference between project success and failure. The stakeholder
register is a project document where information about identified stakeholders is recorded. This document includes the
stakeholders' main requirements and expectations for the project. Failure to appropriately identify key stakeholders early enough
in the project could result in an incomplete stakeholder register and a project with numerous change requests submitted by
stakeholders as they are identified.

Question 38 - Qid 6112019, Quality Management, 2. Process, 8.3 Control Quality

In the design phase, external experts verified that the team's blueprints for a new crane complied with requirements. During the build
phase, the customer performed a walkthrough and found that some welding joints on the crane were defective. The crane needs to be
reworked to ensure it can be operated safely.

How could this situation have been avoided?

Outsourcing the welding to an external vendor


Testing the welding before the customer walkthrough
Using the requirements traceability matrix
Defects are part of any project and cannot be avoided.

Hint Unmarked

Your answer is incorrect


Explanation:

The question describes a situation in which the project team completed the design phase of the project and began building the
product. However, it appears the team did not sufficiently test the work done on the welding, which is part of the Control Quality
process. This process is carried out to ensure the project outputs are complete, correct, and meet customer expectations before
the customer accepts the deliverables as part of the Validate Scope process. As a result of not performing the Control Quality
process, defects were found in the crane when the customer held a walkthrough. If the team carried out the Control Quality
process properly, it is likely the defective welding would have been discovered, corrected, and the situation described in the
scenario avoided.
Question 39 - Qid 6112028, Integration Management, 3. Business Environment, 1.2 Foundational Elements

For an upcoming project, a needs assessment and business case have been completed, and the project management office (PMO) is
reviewing a draft of the benefits management plan. The PMO highlights an element of the benefits management plan and requests that the
element be removed before the document is finalized.

Which of the following was most likely the element the PMO requested to remove?

Expected tangible and intangible value to be gained by the implementation of the project
A set of options to be considered for addressing the business opportunity
A timeframe for realizing the benefits of the project
Measures to be used to show benefits realized

Hint Unmarked

Your answer is incorrect


Explanation:

The key elements of a benefits management plan include target benefits, strategic alignment, a timeframe for realizing benefits,
benefits owner, metrics, assumptions, and risks. Identification of a set of options to be considered for addressing the business
problem or opportunity is a key element of the project business case and should not be included in the benefits management
plan. This element is most likely what was marked for removal by the PMO.

Question 40 - Qid 6113170, Resource Management, 1. People, 9.4 Develop Team

You have been requested to fill in for a project manager who has called in sick midway into project execution. As you get yourself familiar
with the project team, you are impressed to see how well the team members collaborate and work through issues smoothly and effectively.

Based on your observations, where is the team on the Tuckman ladder?

(Please note that on the real PMP exam you may be asked to provide your answer by clicking the correct area in the image. But here in the
simulator, we are asking you to select the answer below.)

A
B
C
D

Hint Unmarked

Your answer is incorrect


Explanation:

The Tuckman ladder is one of the models used to describe stages of team development. This model is typically referred to by
project management practitioners as part of the Develop Team process. The model includes five stages: forming, storming,
norming, performing, and adjourning. While it is safe to say that most teams go through all five stages, some may get stuck in a
particular stage or even regress (go back) to an earlier stage. On the other hand, projects with team members who worked
together in the past might skip a stage. Each stage is distinguished by different dynamics among the team members resulting in
various levels of team effectiveness. Team effectiveness is at its lowest level during the storming stage as the environment is
characterized by a high degree of conflict and a lack of collaboration among the team members. As the team members begin to
work together and trust each other, in other words, the relationships between the team members normalize (thus the name of the
next stage, norming), the degree of team effectiveness increases. Teams that successfully go through the norming stage would
eventually reach the performing stage, which is the highest level of team development. In the performing stage, the team function
as a well-organized unit, and the team effectiveness is at its highest.

Please note that on this page we only show a general explanation for the 120 questions.

The PMP Exam Simulator (http://www.pm-exam-simulator.com/pmp), however, will give you a much richer experience
because you also receive an explanation of why each of the answer choices is correct or incorrect, as well as a reference that
allows you to read up on the subject and learn from any mistakes.

Question 41 - Qid 6112039, Scope Management, 2. Process, 5.3 Define Scope

To define the product that will be produced by the project, the project team is using a product analysis technique that helps with breaking
down the high-level requirements into the level of detail needed to design the product.

Which of the following is the product analysis technique used by the team?

Work breakdown structure


Product planning
Progressive elaboration
Product breakdown

Hint Unmarked

Your answer is incorrect


Explanation:

Product analysis can be used as a tool to define the scope of a project. It involves methods for translating high-level product or
service descriptions into meaningful deliverables. Requirements are captured at a high level and decomposed to the level of
detail needed to design the final product and define the scope. Examples of product analysis techniques include product
breakdown, requirements analysis, systems analysis, systems engineering, value analysis, and value engineering. In the scenario
described, the team breaks down the high-level requirements into the level of detail needed to design the product, which
corresponds to the product breakdown technique, making it the best answer among the choices given.

Question 42 - Qid 6112054, Integration Management, 3. Business Environment, 4.7 Close Project or Phase

A three-phase project to build a prototype is underway. The first phase entails design. In the second phase, the manufacturing team would
produce the parts from the design specifications. In the third phase, the production team would assemble the parts to construct the
prototype. The project is now at the end of the first phase.

What should be done with the phase's deliverables?

Verified by the testing team for quality compliance


Transferred to the production team
Transitioned to the project management office (PMO)
Handed over to the manufacturing team

Hint Unmarked

Your answer is incorrect


Explanation:The question states that the project is at the end of the first phase, implying the Close Project or Phase process is
being carried out. An output of the Close Project or Phase is the transition of the final product, service, or result to a different
group or organization that will operate, maintain, or support it throughout its life cycle. Officially transferring the ownership of
deliverables to the assigned stakeholders facilitates project (or phase) closure. In this case, the project manager would transfer
ownership of the design to the manufacturing team since they are responsible for the second phase of the project.

Question 43 - Qid 6112056, Integration Management, 2. Process, 4.3 Direct and Manage Project Work

A project is nearing its end when the project manager receives an approved change request to replace a defective unit.

What should the project manager do next?

Capture the problem in the issue log


Meet with the change control board
Have the defective unit repaired
Have the defective unit replaced

Hint Unmarked

Your answer is incorrect


Explanation:

The question suggests that the Direct and Manage Project Work process is being carried out. Approved change requests are one of
the inputs to that process. The project manager has received an approved change request. It is now her responsibility to ensure
the approved change request is implemented as specified. Since the change request calls for the replacement of the defective
unit, the next step for the project manager is to have the defective unit replaced.

Question 44 - Qid 6112060, Integration Management, 2. Process, 4.6 Perform Integrated Change Control

A technician has just tested a piece of equipment and found a defect. The technician has informed the project manager and submitted a
change request to repair the defect, indicating that the repair will require significant rework.

What should the project manager do first?

Approve the change request


Consult the change management plan
Instruct the team to repair the defect
Reject the change request due to significant rework

Hint Unmarked

Your answer is incorrect


Explanation:

During the Perform Integrated Change Control process, change requests may be submitted for review and have a decision made
based on the merits of the request. A defect repair requires an approved change request, but the project manager may or may not
have the authority to approve the change request. The change management plan provides direction for managing the change
control process and documents the roles and responsibilities. The project manager should check the change management plan
first to know which individual or group is responsible for reviewing and making decisions on change requests.

Question 45 - Qid 6112061, Integration Management, 2. Process, 4.0 Project Integration Management

At the end of a design project, a competitor has gained market share with a new technology that has rendered the project?s deliverable
obsolete. The project sponsor is pleased with the work of the project team who finished within budget and on time but thinks that the
project should have been terminated much sooner.

What might have the manager of the project done differently in this situation?

Used phases and phase gates


Established a change control board (CCB) to review and render decisions for all submitted change requests
Terminated the project when the new technology was introduced
Scheduled more frequent lessons learned meetings with the project team
Hint Unmarked

Your answer is incorrect


Explanation:

During project integration, a project manager needs to consider how to tailor the way that Project Integration Management
processes are applied. Among these considerations is the project life cycle including the appropriate use of phases and phase
gates. A phase gate is a review at the end of a phase in which a decision is made to continue to the next phase, to continue with
modification, or to end a project. During a phase gate, the business case should be examined to determine if it is still valid or
needs to be modified. In some cases, for example, changes in the competitive landscape may lead to a decision to terminate the
project as the business case has been invalidated. In this scenario, the business case is no longer valid due to new technology,
which was introduced in the marketplace and rendered the current product in development obsolete. Of the available options,
only using a phased approach with phase gates introduces a periodic structured review of the viability of the project, making this
the best answer to the question asked.

Question 46 - Qid 6112064, Schedule Management, 2. Process, 6.0 Project Schedule Management

A project is underway to construct a wind farm in an area with no communication infrastructure. The plan is to monitor progress and
coordinate the scheduling of the project activities using project management software and provide daily status updates to the project
sponsor at the home office. Two project teams will work and reside onsite in rotating seven-day tours.

Which of the following is the most likely challenge the teams will face while controlling the schedule?

Stability of requirements
Technology support
Human resource availability
Project importance

Hint Unmarked

Your answer is incorrect


Explanation:

Each project is unique. Therefore, the project manager may need to tailor the way Project Schedule Management processes are
applied. Technology support is among the considerations for tailoring that should be kept in mind while planning the project
under the circumstances described in the scenario. The question presents a situation where a project is being conducted in a
desolate area without any existing infrastructure for technology support. This circumstance means that there are no cellular
towers or Wi-Fi and no readily available means of communication. Concerning controlling the schedule, the project manager must
consider how the project management software, including the project schedule, will be updated and submitted daily to the home
office. The project manager may need to acquire the necessary equipment to use satellite technology and reconsider the content
and frequency of the project status updates. Therefore, of the available choices, technology support is the best answer to the
question asked.

Question 47 - Qid 6112068, Resource Management, 1. People, 9.5 Manage Team

A project is behind schedule because department managers reassigned project team members to work on other assignments. After
negotiations with the managers, you agreed to a temporary solution where a core group of project resources is dedicated to performing the
project work until more resources are approved.

What conflict resolution technique did you most likely employ in this situation?

Compromise or reconcile
Force or direct
Collaborate or problem solve
Smooth or accommodate

Hint Unmarked

Your answer is incorrect


Explanation:

The question states that a temporary solution was suggested that provided some degree of satisfaction for both managers. The
compromise/reconcile approach to conflict resolution is characterized by searching for solutions that bring some degree of
satisfaction to all parties to temporarily or partially resolve a conflict. Even though the compromise/reconcile approach provides a
lose-lose solution because both parties have to give up something, it is the best answer based on the situation described by the
question.

Question 48 - Qid 6112072, Scope Management, 2. Process, 5.6 Control Scope

During a daily stand-up meeting, one of the programmers mentions that she had some extra time and added functionality, which was not
included in the original design. She believes the customer will appreciate the added functionality.

What is your best course of action?

Remove the added functionality and ensure that the project team builds the software as designed going forward
Thank the programmer for the extra effort and notify the customer of the new functionality that has been added
Update the cost baseline since the project will run over budget due to the cost of adding new functionality
Accept the added functionality, but ask the programmer to build only what has been designed going forward

Hint Unmarked

Your answer is incorrect


Explanation:

The extra code that was written to add the functionality should be removed from the build, and the project manager should
ensure software is written as designed going forward. Adding unplanned functionality, sometimes referred to as gold plating, will
increase the overall project risk. For example, there may have been a good reason the client did not want this added functionality.
Perhaps the client's computer system cannot support more advanced functionality. Additionally, the added code may interfere
with other functionality since it was not integrated with the original build plan. Of the available choices, the best course of action
is to remove the added functionality and ensure that the design plan is followed in the future. Note that if the project manager
believes that this added functionality would benefit the customer, then the suggestion can be made to the customer. The
customer can then decide whether or not to add the functionality and accept any impact on other project constraints. If the
customer approves the added functionality, a change request should be submitted and processed through the Perform Integrated
Change Control process.

Ideally, a risk of removing the functionality should be assessed first including the impact on all project constraints, and then a
decision would have to be made accordingly. However, this answer choice is not provided. Therefore, as we always recommend,
we have to select the best answer of those provided, even if it does not look/sound like the perfect/ideal answer or the answer
one would want to see among the options.

Question 49 - Qid 6112082, Quality Management, 2. Process, 8.2 Manage Quality

The project manager is managing the research and development of a new cancer screening test. The project manager would like to
determine whether the processes currently in use in the project are effective in meeting the quality requirements specified by the customer.

What does the project manager need to obtain first to make this determination?

Quality control measurements


Resource breakdown structure
Requirements traceability matrix
Acceptance of deliverables from the customer

Hint Unmarked

Your answer is incorrect


Explanation:

The scenario implies that the project manager wants to perform Manage Quality which is the process of translating the quality
management plan into executable quality activities that incorporate the organization?s quality policies into the project. Ensuring
that the processes used in the project are effective in meeting the quality requirements needed to satisfy the customer is one of
the goals of the Manage Quality process. However, to do this, the Manage Quality process uses the quality control measurements
gathered during the Control Quality process. Therefore, to determine whether the processes currently in use in the project are
effective in meeting the quality requirements needed to satisfy the customer, the project manager first should obtain the quality
control measurements. Note that typically outputs from the executing processes become inputs to the monitoring and controlling
processes. However, in this case, quality control measurements are an output from the Control Quality process, a monitoring and
controlling process, and an input to the Manage Quality, an executing process.

Question 50 - Qid 6112095, Quality Management, 2. Process, 8.3 Control Quality

During an inspection of a project deliverable, the team detects the same defect in the deliverable that has already been identified during a
previous inspection. The project manager is confused about how this could have happened since a change request was approved to repair
the defect. After discussing the issue with the team, the project manager learns that the team never implemented the approved change
request.

What could have helped prevent this situation?

Holding an approved change requests review


Conducting a retrospective meeting
Performing a root cause analysis
Creating a quality report

Hint Unmarked

Your answer is incorrect


Explanation:

Inspections carried out as part of the Control Quality process may uncover defects or areas of noncompliance with project
requirements, which, in turn, may generate change requests. It is the responsibility of the project team to ensure that those
approved change requests are implemented and properly tested, completed, and certified. In this scenario, the project manager
learns that an approved change request was never implemented. Retrospectives, root cause analysis, and quality reports would
not have prevented this issue but could be used to help the team avoid such a mistake in the future. An approved change request
review would have provided the project manager and the team a mechanism for verifying that the approved change request was
implemented and is, therefore, the best answer to the question asked.

Question 51 - Qid 6112200, Quality Management, 2. Process, 8.1 Plan Quality Management

The team has just reported to you that they have completed the Plan Quality Management process. You review their work and see that they
have produced the quality management plan, and made the necessary updates to the project management plan and project documents.

What did they forget?

Determine quality metrics


Define quality roles and responsibilities
Revise the lessons learned register
Update the requirements traceability matrix

Hint Unmarked

Your answer is incorrect


Explanation:

The question suggests that the Plan Quality Management process was performed, and an element of this process was missed.
Quality metrics are a project document that serves as one of the outputs of the Plan Quality Management process. This document
was not mentioned in the scenario. All of the other outputs are stated as being completed. A quality metric, an input to the
Manage Quality and Control Quality processes, specifically describes a project or product attribute and how these processes will
verify compliance with it. Therefore, quality metrics are what was most likely missed by the project team.

One may argue that quality metrics should be part of the quality management plan just like the quality roles and responsibilities.
While this may sound counter-intuitive, the PMBOK® Guide describes quality metrics as a separate standalone document. The
complete list of the components of the project management plan vs. project documents can be found on page 89, Table 4-1.
Project Management Plan and Project Documents.
Question 52 - Qid 6112235, Integration Management, 2. Process, 4.6 Perform Integrated Change Control

You are managing a project to build a house for a private client. A few weeks before the project's planned completion date, the client calls
and asks to add an outdoor sauna to the property. You analyze the project schedule and determine that this change would be fairly easy to
implement without having an impact on the completion date.

What should you do first?

Begin construction since the project completion date is not violated


Review the requested change against all project constraints
Have the sponsor verify if the change violates construction regulations
Consult with the change control board on how to handle the change

Hint Unmarked

Your answer is incorrect


Explanation:

A project manager is responsible for overseeing the development of the product of the project. This includes managing changes
that arise during execution by balancing competing constraints with the resources available. In the scenario presented, the
project manager is faced with managing a change introduced late in the project life cycle. While it appears that an outdoor sauna
will not be difficult to implement and that no negative schedule impacts are anticipated, the change request must still be
examined in greater detail using the Perform Integrated Change Control process to assess the impact of the change on other
project constraints, such as the the project schedule, costs, quality, resources, risks, etc., and communicate the impact of the
change to the client.

What if the cost of adding the sauna will exceed the budget the client had in mind for the project? What if the project does not
have enough resources to implement the change? These and other questions should be asked when a change request is properly
evaluated. Therefore, of the choices provided, reviewing the requested change against all project constraints is the best answer to
the question asked.

Question 53 - Qid 6112294, Integration Management, 2. Process, 4.1 Develop Project Charter

You have been asked to lead a product development project which will use an agile framework. Currently, you are in the process of drafting
the project charter, and you want to bring together stakeholders and subject matter experts to discuss perceived project risk, success
criteria, and other topics.

What is the best way for you to accomplish this objective?

Facilitate an iteration retrospective


Invite the relevant participants to the daily scrum meetings
Conduct interviews with the identified individuals
Schedule a focus group

Hint Unmarked

Your answer is incorrect


Explanation:

The question states that the project charter is being drafted which implies that the project is in the initiation stage. A focus group
is a data-gathering technique which may be beneficial in the development of the project charter. Focus groups bring together
stakeholders and subject matter experts to learn about the perceived project risk, success criteria, and other topics in a more
conversational way than a one-on-one interview. The expectations of the project manager do not change in an adaptive (agile)
environment, but control of the detailed product planning and delivery is delegated to the team who, in the scenario described,
can be considered the subject matter experts. The incorrect answer choices represent meetings that would take place after
project initiation has been completed or are not held in a group setting. Therefore, of the available choices, scheduling a focus
group is the best answer.
Question 54 - Qid 6112434, Procurement Management, 2. Process, 4.7 Close Project or Phase

A multi-phase project has reached a phase gate. The key project stakeholders want to determine if the phase has met its success criteria
and if the project should proceed to the next phase.

In addition to the project management plan, which documents will be required?

Risk register and risk report


Lessons learned register and lessons learned repository
Business documents and project charter
Agreements including procurement contracts

Hint Unmarked

Your answer is incorrect


Explanation:

A phase gate is a review point at the end of a phase to determine if the project will continue to the succeeding phase, end the
project, repeat the phase, or continue with some modifications. The documents required to make this determination are the
project charter, the business case, and the benefits management plan. The last two are collectively known as the business
documents. The project charter provides the project's success criteria. The business case provides information about the business
need and the cost-benefit analysis that justifies the project. The benefits management plan is used to measure whether the
benefits of the project are being achieved as planned. Therefore, to determine if the phase has met its success criteria and if the
project should proceed to the next phase, in addition to the project management plan, one would need the project charter and
business documents.

Question 55 - Qid 6112456, Integration Management, 2. Process, 4.4 Manage Project Knowledge

You are in the process of establishing a knowledge management system for your project.

Which of the following actions will be the least beneficial in this process?

Reviewing the lesson learned register


Monitoring stakeholder engagement
Examining the resource breakdown structure
Studying the project team assignments

Hint Unmarked

Your answer is incorrect


Explanation:

Central to knowledge management is having the right processes and people with the right knowledge. Having documents that
identify the people and their capabilities are essential to identifying current knowledge and the gaps that may exist. Keep in mind
that the question is asking for the action that will be 'least' beneficial in this process.

Monitor Stakeholder Engagement is the process of monitoring stakeholder relationships and tailoring strategies for engaging
stakeholders through the modification of engagement strategies and plans. This process helps maintain or increase the efficiency
and effectiveness of stakeholder engagement activities as the project evolves and its environment changes. The Monitor
Stakeholder Engagement process belongs to the Monitoring and Controlling Process Group, while according to the scenario, the
project manager is carrying out the Manage Project knowledge process, which is part of the Executing Process Group.

Monitoring stakeholder engagement does not help identify the people or processes needed to establish a knowledge
management system for the project, whereas the other options, namely the lessons learned register, the resource breakdown
structure, and project team assignments are useful in determining what is required for a knowledge management system.
Therefore, of the available choices, monitoring stakeholder engagement would be the least beneficial for establishing a
knowledge management system for the project and is, therefore, the best answer to the question asked.
Question 56 - Qid 6112463, Stakeholder Management, 2. Process, 13.3 Manage Stakeholder Engagement

You are leading a large project that involves many stakeholders. You want to get two new team members up to speed with the overall
dynamics of the project and engage effectively with the project stakeholders.

What should you do?

Ask the team members to record any issues they encounter into the issue log
Review the stakeholder register with the new team members
Schedule a meeting with the new team members and all project stakeholders
Submit a change request to update the stakeholder engagement plan

Hint Unmarked

Your answer is incorrect


Explanation:

The stakeholder register is a project document that contains all current information about the stakeholders. The stakeholder
register includes the stakeholder identification information, assessment information, and stakeholder classification. It is used by
the project team to maintain information about the stakeholder's expectations and interests with the project. By reviewing the
stakeholder register with the new team members, the project manager can provide them with the information about each project
stakeholder, politics surrounding the project, and other relevant details about the project and the stakeholders that may help the
new team members to get up to speed with the overall dynamics of the project and engage effectively with the project
stakeholders.

Question 57 - Qid 6112493, Resource Management, 1. People, 9.1 Plan Resource Management

You are facilitating a meeting with your project team to develop the team charter. At the start of the meeting, a team member states, "I
don't understand the importance of this meeting. We are all professionals, and this isn't our first project working together. Wouldn't our
time be better spent getting an early start on project execution?" The body language of the other team members suggests that they agree
with the comments.

How should you respond to the team member?

As long as everyone is in agreement, then let's skip the team charter and get ahead of schedule
Let's use the team charter from our last project as a starting point and then update it as necessary
The team charter is required by the project management office (PMO) for all projects
The team charter will eliminate any potential conflicts which will improve our productivity

Hint Unmarked

Your answer is incorrect


Explanation:

The team charter establishes clear expectations regarding acceptable behavior by the project team members. An early
commitment to clear guidelines decreases misunderstandings and increases productivity. The question indicates that the project
team has worked together on previous projects. If the team charter from the last project was effective, it serves as a good starting
point for use on the current project. It is essential that the project manager gain the commitment of the project team for the
guidelines established by the team charter, and if using the previous team charter as a template helps gain the commitment of the
project team, then it is a worthwhile compromise. Of the available choices, using the previous team charter as a starting point
provides the best opportunity to gain the commitment of the project team.

Question 58 - Qid 6112596, Procurement Management, 2. Process, 12.1 Plan Procurement Management

You are leading a project to build an oil pipeline in a foreign country. You want to outsource some of the construction work to local
contractors and are in the process of preparing the bid package for prospective sellers.

Which of the following documents would you leave out of the package?

Request for proposal


Statement of work
Independent cost estimates
Bid documents

Hint Unmarked

Your answer is incorrect


Explanation:

Preparing the bid package for prospective sellers implies you are carrying out the Plan Procurement Management process. Bid
documents along with the request for proposal (RFP), procurement statement of work (SOW), source selection criteria,
independent cost estimates are among the outputs of this process. These documents are also among the inputs to the Conduct
Procurements process. However, not all of these documents are provided by the buyer to the prospective sellers. Sometimes, to
benchmark procurements, the buying organization develops independent cost estimates either internally or using external
resources such as a professional estimator. Buyers do not usually provide these estimates to the prospective sellers since a
benchmark range may prejudice the sellers into proposing prices that do not work well with their business model and resources.
The buyer uses these benchmark estimates principally to verify if bids are reasonable, or if a large number of outliers indicate an
issue with procurement processes or communications. Thus, of the choices provided, leaving the independent cost estimates out
of the bid package makes the most sense and is, therefore, the best answer to the question asked.

Question 59 - Qid 6112605, Communications Management, 2. Process, 10.1 Plan Communications Management

You are developing the communications management plan for a construction project. You want to determine the information needs of the
project stakeholders, specifically focusing on the type and the format of information that will be required to communicate with the
stakeholders.

What is your best course of action?

Develop the stakeholder register


Review the requirements documentation
Conduct a communication requirements analysis
Consult the stakeholder engagement plan

Hint Unmarked

Your answer is incorrect


Explanation:

The question implies that you are performing the Plan Communications Management process with the development of the
communications management plan. Communication requirements analysis is one of the tools and techniques that may be used
in this process. Analysis of communication requirements determines the information needs of the project stakeholders. These
requirements are defined by combining the type and format of information needed with an analysis of the value of that
information. Therefore, of the available answer choices, conducting a communication requirements analysis is the best course of
action in this scenario.

Question 60 - Qid 6125134, Cost Management, 2. Process, 7.4 Control Costs

The following Gantt chart displays the schedule for a software upgrade project. The system analysis was completed on August 12th, but the
system design and development activities are still in process as of October 3rd. Once the product is developed, the project team will need to
test and deploy it, which must be completed by October 17th. The method used to determine the earned value is based on percent
complete.

What is the cost variance as of October 3rd?


-$650
$3,750
$650
$3,100

Hint Unmarked

Your answer is incorrect


Explanation:

Cost variance (CV) is the earned value (EV) minus the actual costs (AC). The current earned value is the sum of the earned values
for every activity that has either been completed or is in progress. Thus, the earned value can be calculated by multiplying the
activity's percent complete by the activity's planned value. The actual costs are the total costs of individual activities. Note, since
the scenario mentions that the method used to determine the earned value is based on percent complete, the earned value of a
partially completed work package is calculated by multiplying the planned value of the work package by its the percent complete.

The calculations are shown below:

EV = EV of activity 1 + EV of activity 2 + EV of activity 3 = (1 x $500) + (0.75 x $1,000) + (0.50 x $5,000) = $3,750

AC = AC of activity 1 + AC of activity 2 + AC of activity 3 = $600 + $500 + $2,000 = $3,100

CV = EV - AC = $3,750 - $3,100 = $650

Therefore, the cost variance (CV) in this scenario is $650. Since the CV is a positive value, the project is running under planned
costs. Note, the information about testing and deployment that must be completed by October 17th is irrelevant for selecting the
correct answer.

Question 61 - Qid 6112614, Resource Management, 1. People, 9.0 Project Resource Management

A servant leader has just been assigned to an upcoming product development project and believes that the project is well-suited to
Kanban. However, the development team is unfamiliar with this approach.

What is the servant leader's best course of action?

Discuss Kanban during a daily standup meeting


Review Kanban at the iteration retrospective
Let the team members learn Kanban on their own
Have the team attend a workshop on Kanban

Hint Unmarked

Your answer is incorrect


Explanation:

The scenario implies that the servant leader wants to use a project management approach with which the development team is
unfamiliar. If the servant leader believes that the Kanban approach is the best methodology for the project, then the servant
leader has the responsibility to ensure that the development team has the appropriate training to execute the project. Of the
available options, conducting a Kanban workshop would be consistent with agile best practices and provide the development
with the necessary tools to execute the project.

Question 62 - Qid 6112667, Scope Management, 2. Process, 5.3 Define Scope

You are developing the project scope statement. You have specified the deliverables for the project, included the requirements under which
the deliverables will be accepted, and explicitly stated what is out of project scope.

What else should you include in the project scope statement?

Project exclusions
Acceptance criteria
Project approval requirements
Product scope description
Hint Unmarked

Your answer is incorrect


Explanation:

The scenario describes the development of the project scope statement which is an output of the Define Scope process. The
project scope statement is the description of the project scope, major deliverables, assumptions, and constraints. The entire
scope, including project and product scope, is documented in the scope statement. The detailed project scope statement includes
the following:

Product scope description which progressively elaborates the characteristics of the product, service, or result described in
the project charter and requirements documentation.
Deliverables which describe any unique and verifiable product, result, or capability to perform a service that is required to be
produced to complete a process, phase, or project.
Acceptance criteria which are a set of conditions that is required to be met before deliverables are accepted.
Project exclusions which explicitly identify what is excluded from the project.

According to the scenario, of the four elements listed above, three have already been specified in the project scope statement. The
only missing element is the product scope description.

Note, acceptance criteria included in the project scope statement are at the deliverable level. The WBS dictionary (which is not the
topic of this question) includes acceptance criteria for each individual work package specified in the WBS.

Question 63 - Qid 6112676, Resource Management, 2. Process, 9.3 Acquire Resources

A project manager is in the process of obtaining team members. She wants to understand the availability of the team members required to
complete project work.

What should the project manager do to find this information?

Look at the resource calendars


Contact the project management office
Review the resource requirements
Examine the resource breakdown structure

Hint Unmarked

Your answer is incorrect


Explanation:

The scenario implies that the project manager is about to start one of the iterations of the Acquire Resources process and needs to
understand the availability of the project team members. The project documents include the resource calendars which serve as
an input to the Acquire Resources process. A resource calendar identifies the working days, shifts, start and end of normal
business hours, weekends, and public holidays when each specific resource is available. Information on which resources are
potentially available during a planned activity period is used for estimating resource utilization. Resource calendars also specify
when and for how long the identified team and physical resources will be available during the project. Of the available choices,
looking at the resource calendars provides the best source of information for the project manager about the availability of the
team members to perform project work.

Note, resource calendars are progressively elaborated and updated throughout the project. Once created as an output of the
Acquire Resources process, they are used as needed whenever this process is repeated. In other words, after the Acquire
Resources process is completed for the first time, resources calendars become an input to each subsequent iteration of the
process, which is the assumption the scenario described.

Question 64 - Qid 6112685, Quality Management, 2. Process, 8.2 Manage Quality

A project to reduce the defect rate of the manufacturing process is underway. To analyze and improve the process, the project team is now
carrying out the quality assurance activities.

Which of the following would be the most beneficial for this project?
Introducing the use of a kanban board into the manufacturing process
Consulting with a certified Six Sigma Black Belt expert
Utilizing rolling wave planning for the project
Adopting a just-in-time (JIT) manufacturing process

Hint Unmarked

Your answer is incorrect


Explanation:

The question implies that the project manager is carrying the Manage Quality process. Quality improvements can occur based on
findings and recommendations from the quality control processes, the findings of the quality audits, or problem-solving in the
Manage Quality process. Plan-do-check-act and Six Sigma are two of the most common quality improvement tools used to
analyze and evaluation opportunities for improvement. A person with the Six Sigma Black Belt certification is an individual that
has been trained to apply Six Sigma tools and techniques for process improvement. Six Sigma improves quality by identifying and
eliminating the causes of defects and minimizing the variability in a manufacturing process. A Six Sigma process is one in which
99.99966% of the products of the process are free of defects. Of the available choices, only consulting with a certified Six Sigma
Black Belt expert would be beneficial in reducing the defect rate of a manufacturing process.

Question 65 - Qid 6112687, Risk Management, 2. Process, 11.6 Implement Risk Responses

During project planning of a large, complex project, project management best practices were rigorously and thoroughly followed. During
project execution, the project experienced several critical risks, which were foreseen and previously identified. Despite the meticulous
planning, the project failed to achieve the established objectives due to these realized risks.

What may have gone wrong in this scenario?

The realized risks had not been identified earlier in the project
The risk responses were poorly devised
The qualitative risk analysis was inadequate
There was little or no action taken to manage the known risks

Hint Unmarked

Your answer is incorrect


Explanation:

Proper attention to the Implement Risk Responses process will ensure that the agreed-upon risk responses are actually executed.
A common problem with Project Risk Management is that project teams spend effort in identifying and analyzing risks and
developing risk responses, then risk responses are agreed upon and documented in the risk register and risk report, but no action
is taken to manage the risk. The incorrect answer choices all represent activities that occurred during project planning, which
according to the scenario, was meticulous and followed all project management best practices. Therefore, the most likely reason
for the project failing to meet objectives is the result of inadequate attention to the Implement Risk Responses process during
project execution. Thus, among the available choices, taking little or no action to manage the known risks is the best answer to
the question asked.

Question 66 - Qid 6112688, Communications Management, 1. People, 10.2 Manage Communications

A team member volunteered to provide the project status update in a weekly presentation to the key stakeholders. After the first meeting
has taken place, feedback from the stakeholders suggests that the presentation was not very engaging.

What is the best way for the project manager to address this situation with the team member?

Discuss the issue with the team member


Prepare a formal written document
Utilize formal verbal communication
Send an email to the team member

Hint Unmarked
Your answer is incorrect
Explanation:

The project manager has to decide on the appropriate choice of media to use while addressing the situation with the team
member. The choice of media includes formal versus informal communication and written versus verbal communication. There is
nothing in the scenario to suggest that the situation is recurring or severe warranting formal communication (either written or
verbal). The team member has volunteered. Having an informal discussion with the team member will allow the project manager
to set appropriate expectations in a personal and less threatening manner than the other forms of communication. Such a
discussion is an example of informal verbal communication.

Note, the correct answer choice, "discuss the issue with the team member", does not specify whether the discussion is formal or
informal. However, it is reasonable to assume that such a discussion would be informal. Regardless, whether the discussion is
formal or informal, of the choices provided, discussing the issue with the team member is the best answer to the question asked.

Question 67 - Qid 6112694, Stakeholder Management, 2. Process, 13.3 Manage Stakeholder Engagement

A corporate restructuring project is in execution. The job responsibilities of many key stakeholders are being disrupted, which has caused
challenges in keeping the stakeholders engaged in a positive manner.

Which of the following actions is the project manager least likely to perform in this situation?

Manage stakeholder expectations through negotiation and communication


Develop approaches to involve stakeholders based on their needs, expectations, interests, and potential impact on the
project
Address any risks or potential concerns related to stakeholder management and anticipate future issues that may be
raised by stakeholders
Engage stakeholders at appropriate project stages to obtain, confirm, or maintain their continued commitment to the
success of the project

Hint Unmarked

Your answer is incorrect


Explanation:

The question implies that the project manager is implementing the stakeholder engagement plan, which suggests that the
Manage Stakeholder Engagement process is being carried out as part of project execution. The development of approaches to
involve stakeholders based on their needs, expectations, interests, and potential impact on the project describes the
development of the stakeholder engagement plan, which is performed during project planning rather than project execution.
Note, the question is asking for the activity, which is 'least' likely to be performed in this situation. Therefore, of the available
choices, the project manager is 'least' likely to be developing the stakeholder engagement plan in this scenario.

Question 68 - Qid 6112702, Resource Management, 2. Process, 9.6 Control Resources

You are leading a software development project where the developers are colocated within rented office space. Writing the code is taking
longer than anticipated, and you have contacted the landlord requesting to rent the office for another week. Unfortunately, the landlord
tells you that the office is already booked. You capture the problem in the project documents.

What should you do next?

Record the issue in the issue log


Revise the resource management plan
Update the risk register
Request additional funds

Hint Unmarked

Your answer is incorrect


Explanation:
The office is a physical project resource. Contacting the landlord requesting to rent the office for another week is a function of the
Control Resources process. Control Resources is the process of ensuring that the physical resources assigned and allocated to the
project are available as planned, as well as monitoring the planned versus actual utilization of resources and performing
corrective action as necessary. In this case, the actual utilization of the office space will exceed the planned utilization. The
resource management plan is a component of the project management plan that describes how project resources are acquired,
allocated, monitored, and controlled. Updates to the resource management plan are an output of the Control Resources process.
The plan may be updated to reflect actual experience in managing project resources. Therefore, among the available options,
revising the resource management plan is what you should do next.

Note, an approved change request will be required to update the resource management plan as it is a component of the project
management plan.

Question 69 - Qid 6112760, Procurement Management, 2. Process, 12.2 Conduct Procurements

During a bidder conference, a supplier asks why a section is missing from the request for quotation (RFQ). Upon realizing that the section is
indeed missing, the project manager becomes flustered and stumbles through a brief reply. When the seller responses were received, all but
three prospective suppliers fail to address the missing section.

What is the best course of action?

Hold the bidder conference again, and explain the importance of the point that was initially missed
Select from the three vendors that submitted complete proposals
Send a revised RFQ to all prospective suppliers and allow them the opportunity to resubmit their proposals
Extend the deadline and allow all of the vendors to resubmit their proposals

Hint Unmarked

Your answer is incorrect


Explanation:

In this scenario, sending the new RFQ to all suppliers, clarifying the critical point that was previously missing, and receiving
updated responses to the RFQ would be fair to all suppliers. It may not be practical or cost-effective to conduct another bidder
conference, especially as other aspects of the project have already been covered in the prior bidding conference. Apart from
fairness, re-issuing the RFQ ensures that requirements are understood. Another benefit of sending out a revised RFQ would be
receiving more viable bids from other vendors. Therefore, among the available options, the best course of action for the project
manager is to send a revised RFQ containing the new information to all prospective suppliers and allow them the opportunity to
resubmit their proposals.

Question 70 - Qid 6112783, Schedule Management, 2. Process, 6.6 Control Schedule

You are leading a project to develop a new web application. Together with your project team you reprioritize the product backlog,
determine velocity for the past iterations, and adapt your work plans accordingly. Additionally, you facilitate retrospectives every two
weeks.

Which of the following processes describes the work you are performing?

Monitor Risks
Manage Quality
Control Schedule
Develop Schedule

Hint Unmarked

Your answer is incorrect


Explanation:

Adaptive life cycles, such as the agile approach, use short cycles to undertake work, review the results, and adapt as necessary.
These cycles provide rapid feedback on the work approaches and suitability of deliverables, and generally involve iterative
scheduling. The project described in the scenario is using an agile approach as evidenced by the use of the retrospectives,
backlog, and iterations. When an agile approach is used, Control Schedule is concerned with conducting retrospectives to correct
and improve processes, reprioritizing the backlog, determining the velocity in each iteration, and adjusting the plans accordingly.
Therefore, of the choices provided, the work you are performing is best described by the Control Schedule process.

Question 71 - Qid 6112785, Schedule Management, 2. Process, 6.1 Plan Schedule Management

You are part of a cross-functional development team piloting an adaptive approach to project management in an organization that has
traditionally used a predictive approach. You realize that you have to tailor the template of the schedule management plan.

Which of the following components of the template are you likely to tailor the most?

The rules of performance measurement, the summary milestones, and the key deliverables
The organizational procedures links, the units of measure, and the project organization charts
The schedule model development, the release and iteration length, and the reporting formats
The project schedule model maintenance, the level of accuracy, and the level of precision

Hint Unmarked

Your answer is incorrect


Explanation:

The question suggests you are carrying out the Plan Schedule Management process. The schedule management plan is the
primary output of this process. The schedule management plan establishes the criteria and the activities for developing,
monitoring, and controlling the schedule. Of the choices presented, only the schedule model development, the release and
iteration length, and the reporting formats are components of the schedule management plan. All three of these components
would merit significant tailoring to reflect an adaptive approach to project management. The project schedule model
development specifies the scheduling methodology and the scheduling tool used to develop the schedule model. For an adaptive
approach, the schedule model will be a selected series of activities needed to complete a high-priority subset of the project scope
that can deliver value quickly. The release and iteration length determine the time-boxed events that must be completed and how
long it will take to release the iteration. The reporting formats will be driven by the adaptive schedule model and the iteration
length.

The other answer choices contain components that are not part of the schedule management plan.

Question 72 - Qid 6112817, Risk Management, 2. Process, 11.7 Monitor Risks

A project team performs monthly risk audits for a project, where a large number of identified risks have been realized. So far, the risk
responses have been appropriate, and the reserves sufficient. An executive for the requesting organization criticizes the project manager
improper risk audits, stating that only independent, external resources should perform risk audits.

How should the project manager respond?

Explain that risk audits can be performed either internally or externally as long as they follow the project management
plan
Agree with the executive and submit a change request to update the project management plan to have the audits
conducted externally
Follow the guidance provided by the executive and hire a team of external auditors to conduct the risk audits going
forward
Explain that if the current risk audits are not sufficient, then a comprehensive project audit should be carried out

Hint Unmarked

Your answer is incorrect


Explanation:

Risk audits are among the tools and techniques of the Monitor Risks process. Risk audits are used to consider the effectiveness of
the risk management process. The project manager is responsible for ensuring that risk audits are performed at an appropriate
frequency as defined in the project's risk management plan. Risk audits are typically performed by the project team and may be
included during routine project review meetings or may form part of a risk review meeting, or the team may choose to hold
separate risk audit meetings. In this scenario, since conducting risk audits internally is not contrary to project management best
practices, and the current risk audit process has proven to be effective, there is no reason to agree with the executive. Therefore,
among the available options, the best course of action is to explain to the executive that risk audits can be performed either
internally or externally as long as they follow the project management plan.

Note that any stakeholder can request a change to the project. It is not clear from the question if the executive's comment
constitutes a change request. If, after explaining the rationale for continuing with the current internal risk audits, the executive
still wants external auditors performing the risk audits, the project manager should follow the change control process established
for the project. However, in this situation, the change request would likely be rejected since there is no reason to believe that the
project would benefit from hiring external auditors to perform the risk audits.

Question 73 - Qid 6112819, Risk Management, 2. Process, 11.1 Plan Risk Management

You are developing the risk management plan for your project. In support of this process, you need to determine the acceptable level of
overall project risk exposure.

What should you do first?

Determine the risk appetites of key project stakeholders


Consult the risk exposure register within the organizational process assets
Facilitate a brainstorming meeting to develop a comprehensive list of project risks
Develop a set of responses to individual project risks to mitigate overall project risk

Hint Unmarked

Your answer is incorrect


Explanation:

The risk management plan is created as an output of the Plan Risk Management process. Stakeholder risk appetite is one of the
elements which may be included in the risk management plan. The risk appetites of key stakeholders on the project are recorded
in the risk management plan. In particular, stakeholder risk appetite should be expressed as measurable risk thresholds around
each project objective. These thresholds will determine the acceptable level of overall project risk exposure. The incorrect answer
choices represent activities which would not likely be conducted as part of the Plan Risk Management process. Therefore, among
the available options, determining the risk appetites of key stakeholders is the best answer to the question asked.

Question 74 - Qid 6112895, Integration Management, 2. Process, 4.3 Direct and Manage Project Work

You meet with your team to determine the life cycle for your project. After analyzing the best way to define and manage requirements,
develop deliverables, handle changes, control risk and cost, and engage key stakeholders, the decision is made to select a hybrid life cycle.

With the project life cycle selected, how will the product be delivered?

As subsets of the overall product


As a single final product at the end of the project
As work packages of the WBS
As product increments based on customer's value

Hint Unmarked

Your answer is incorrect


Explanation:

Predictive and agile project life cycles differ from one another in several aspects. One of them is the way the project product,
service, or result is delivered. Predictive projects develop plans up-front and deliver only a single final product (service or result)
at the end of the project. Agile projects, on the other hand, deliver working product increments as frequently as possible based on
the highest value to the customer. Everything in between is defined as a hybrid delivery. With hybrid project life cycles, the
product can be divided into subsets, which are delivered at either pre-defined intervals or as soon as the subsets are completed.
Therefore, of the choices provided, with the hybrid life cycle selected, delivering the result of the project as subsets of the final
product constitutes the best answer to the question asked.
Question 75 - Qid 6112983, Resource Management, 1. People, 9.2 Estimate Activity Resources

As a project manager, you are in the process of estimating the team resources that will be required for your project. The challenge you are
having is that, due to the nature of the project and the selected development approach, only a few of the top levels of the WBS have been
identified with the remainder to be progressively elaborated throughout the project.

What is the best approach to determine the team resource requirements for the project?

Check the organizational process assets for similar projects and use analogous estimating
Apply bottom-up estimating using the work breakdown structure (WBS) as a guide
Utilize parametric estimating for the individual work packages within the existing WBS
Decompose the WBS down to the work package level during initial project planning

Hint Unmarked

Your answer is incorrect


Explanation:

The question suggests that you, as the project manager, are performing the Estimate Activity Resources process and that a hybrid
development approach is being utilized. When incorporating agile practices within an overarching traditional waterfall approach,
the project scope may be progressively elaborated throughout the project, and the work breakdown structure (WBS) may not be
fully decomposed during initial project planning. Under such circumstances, one viable approach to estimating team resource
requirements would be to utilize the analogous estimating technique. If there are past similar projects, the organizational process
assets (OPAs) should contain detailed information that could be used for analogous estimating.

Question 76 - Qid 6112986, Scope Management, 2. Process, 5.4 Create WBS

You are part of a project team designing a new type of a vehicle. You are about to begin work on a work package for the braking system of
the vehicle and want to review the acceptance criteria before starting the work.

Where can you find this information?

Project management plan


Activity attributes
Requirements documentation
The WBS dictionary

Hint Unmarked

Your answer is incorrect


Explanation:

A work package is the lowest level of the WBS and provides a general description of the work to be completed by this work
package. The WBS dictionary supplements the WBS by including more detailed information for each work package or other WBS
components. The WBS dictionary is an output of the Create WBS process and provides information such as the description of
work, schedule milestones, quality requirements, and acceptance criteria for each work package. Therefore, the acceptance
criteria for the work package to be completed are described in the WBS dictionary.

Note, the wording of the question, "You are about to begin work on a work package...", suggests that project execution has already
begun, thus implying the project management plan including all its components (such as the scope baseline that includes the
WBS dictionary) has been finalized and approved.

Question 77 - Qid 6112994, Integration Management, 2. Process, 4.7 Close Project or Phase

A project to construct an assembly line is nearly complete. The project manager closes procurements, shares the final project report,
updates the lessons learned repository, and obtains feedback from the relevant stakeholders.

What should the project manager do next?

Provide final payment to the suppliers


Send a summary of how the final product achieved the business needs
Transfer the completed assembly line to operations
Send formal written notice to the vendors that the contracts have been completed

Hint Unmarked

Your answer is incorrect


Explanation:

The question implies that the Close Project or Phase process is underway. Actions and activities necessary to transfer the project's
products, services, or results to the next phase or production and/or operations is one aspect of project closure. Correctly
answering this question is an exercise in the process of elimination. The incorrect answer choices represent activities that have
already been completed by the project manager in this scenario. Among the available choices, only transferring the completed
assembly line to operations represents a closing activity which has yet to be completed, making it the best response to the
question asked.

Question 78 - Qid 6113034, Risk Management, 2. Process, 11.5 Plan Risk Responses

A project team is mulling over two risk responses for inclusion in the risk register. The first risk response has two criteria that must be met
and costs $5,000. The second risk response has four criteria that must be met, but only costs $2,000.

What action is the project team least likely to take in selecting the best risk response?

Running a Monte Carlo simulation


Performing an alternatives analysis
Conducting a cost-benefit analysis
Using multicriteria decision analysis

Hint Unmarked

Your answer is incorrect


Explanation:

In this scenario, the project team is deciding between two different risk responses for inclusion in the risk register, which suggests
that the project team is currently performing the Plan Risk Responses process. Alternatives analysis, cost-benefit analysis, and
multicriteria decision analysis are all techniques which are commonly used during the Plan Risk Responses process to select risk
responses. Note, the question is asking for the action that the project team is 'least' likely to take in selecting the risk response. A
Monte Carlo simulation is a data analysis technique where a computer model is iterated many times, with the input values chosen
at random for each iteration driven by the input data, including probability distributions and probabilistic branches. Outputs are
generated to represent the range of possible outcomes for the project. However, in this case, there are only two options under
consideration which can serve as inputs for the simulation. Thus, a Monte Carlo simulation would only be able to perform two
unique iterations rather than the many iterations required to produce a probability distribution. Using a Monte Carlo simulation is
not appropriate when only a small, finite number of options are under consideration. Since there are only two options under
consideration, in this case, a Monte Carlo simulation is not the appropriate tool for the project team to use in selecting the best
risk response.

Question 79 - Qid 6113042, Resource Management, 2. Process, 9.3 Acquire Resources

To acquire resources for the project, the project manager consults the resource management plan. The plan calls for the scrum master to be
assigned to the team. However, the only individual in the company qualified to serve as a scrum master has already started working on
another project, and her functional manager does not cooperate on a potential resolution.

How could this situation have been prevented?

The project manager should have checked the project's resource calendars for the availability of the scrum master.
The project sponsor should have directed the project manager to serve as both the project manager and the scrum
master.
The scrum master should have refused to work on the other project knowing that she is the only qualified resource in the
company.
The availability of the scrum master should have been negotiated between the functional manager and the project
manager during project initiation.
Hint Unmarked

Your answer is incorrect


Explanation:

The question implies that the project manager is carrying out the Acquire Resources process. In situations where a key resource is
required for the success of the project, that resource can be-preassigned to the project in the project charter or during initial
project planning. In the scenario, the scrum master was assigned to a different project, and the functional leader is refusing to find
a suitable resolution. Had the project manager negotiated with the foundational manager for the resource ahead of time, the
resource might have still been available. Note that it might be a little unusual for a project to have a project manager and a scrum
master, but it might be necessary for a complex project that will utilize a hybrid project framework.

Question 80 - Qid 6113179, Integration Management, 2. Process, 1.2 Foundational Elements

A project team is using the Stacey Complexity Model to map the degree of uncertainty in the project and select the appropriate project life
cycle. As the meeting progresses, it becomes evident that while the requirements are well-known, the team is unfamiliar with the
technology required to implement them. As a result, the team selects an adaptive project life cycle.

Where on the Stacey Complexity Model did the team most likely map the project?

(Please note that on the real PMP exam you may be asked to provide your answer by clicking the correct area in the image. But here in the
simulator, we are asking you to select the answer below.)

A
B
C
D

Hint Unmarked

Your answer is incorrect


Explanation:

A project life cycle is the series of phases that a project passes through from its start to its completion. (Note, the word "phase" in
this context is used differently from how it's used in multi-phase projects). The project life cycle provides the basic framework for
managing the project. Predictive, iterative, incremental, adaptive, hybrid are examples of a project life cycle. The selection of the
life cycle depends on project characteristics. There are various models that can be used to describe project characteristics. One of
them is the Stacey Complexity Model. The model addresses the degree of uncertainty as it relates to the project requirements on
the one hand and how to fulfill those requirements using current knowledge and technology on the other hand. The technical
degree of uncertainty is mapped on the horizontal axis, and the uncertainty of requirements is on the vertical axis. According to
the model, as project uncertainty increases along any of those dimensions, so too does the risk of rework and the need to use a
different project management approach.

When project requirements, as well as the technology to implement these requirements, are clear, the predictive project life cycle
is typically the way to go. As the degree of uncertainty increases, the likelihood of changes, risk, and rework increases as well.
Such projects would benefit from either iterative, incremental, adaptive, or hybrid life cycles. When both the technical degree of
uncertainty and the degree of uncertainty in project requirements is very high, a project is considered chaos (or anarchy).
Managing such a project is very hard to impossible.

In the scenario, the project requirements are well-known, meaning the degree of uncertainty in requirements is low. However, the
team is unfamiliar with the technology required to implement them, implying the technical degree of uncertainty is high. Based
on these findings, the team selects an adaptive project life cycle, implying the project has been mapped within area C.

Please note that on this page we only show a general explanation for the 120 questions.

The PMP Exam Simulator (http://www.pm-exam-simulator.com/pmp), however, will give you a much richer experience
because you also receive an explanation of why each of the answer choices is correct or incorrect, as well as a reference that
allows you to read up on the subject and learn from any mistakes.

Question 81 - Qid 6113060, Quality Management, 2. Process, 8.3 Control Quality

Your project team members are writing software code together as a pair and reviewing each other's work to determine if the code conforms
to the requirements documentation.

Which of the following will be a result of these reviews?

Accepted deliverable
Acceptance criteria
Work performance data
Verified deliverable

Hint Unmarked

Your answer is incorrect


Explanation:

Writing software code together as a pair and reviewing each other's work is known as pair programming. This practice is often
used on agile projects. As far as traditional project management is concerned, the scenario suggests the team is in the Control
Quality process and is using the tool of inspection, such as peer reviews, to determine if the software conforms to documented
standards. The goal of the Control Quality process and the inspection is to verify the correctness of deliverables thereby producing
the output of verified deliverables. The verified deliverables then become an input to the Validate Scope process to determine if
the deliverables meet the acceptance criteria for formal acceptance by the customer or sponsor. Formal acceptance of the verified
deliverables produces an output from the Validate Scope process called accepted deliverables. Since the team is conducting a
code review to determine the correctness of the software, the result of their efforts will be a verified deliverable.

Question 82 - Qid 6113063, Resource Management, 1. People, 9.5 Manage Team

As part of the Direct and Manage Project Work process, the project manager conducts an iteration planning meeting. The meeting is
dominated by the most senior developer who frequently and sharply interrupts the other team members. The project manager wants to
ensure that during the next iteration planning meeting all voices are heard.

What is the project manager's best course of action?

Discuss the issue with the senior developer, and for the next meeting allocate equal amounts of time for each team
member to speak
Inform the senior developer's functional manager about this unacceptable behavior and request that the manager take
disciplinary action
Post a copy of the team's social contract in the meeting room and review the contents at the start of the next iteration
planning meeting
Invite the sponsor to the next iteration planning meeting to have the senior developer be more thoughtful in the presence
of the sponsor

Hint Unmarked

Your answer is incorrect


Explanation:

The scenario describes a situation where one project team member is dominating a meeting, and not all voices are being heard.
The team charter should include meeting guidelines, including such things as not interrupting and allowing all project team
members to have a voice. In this case, the senior developer may not even realize the disruption that he/she is creating and the
impact on the other project team members. One way of addressing the issue would be to remind the project team members about
their commitments by posting a copy of the team charter and reviewing it before the meeting. The incorrect answer choices
represent actions that are either an overreaction or would not likely be effective in addressing the issue.

It should be noted that there are many ways of addressing the issue described in the question. The project manager probably
should have been more proactive during the meeting to ensure that all voices are heard. Alternatively, the project manager might
have spoken to the senior developer in private about his behavior to address the issue. Regardless, of the available options,
posting the team charter, and reviewing the contents before the iteration planning meeting is the 'best' answer.

Question 83 - Qid 6113070, Scope Management, 1. People, 5.0 Project Scope Management

The scrum master, business analyst, programmers, and testers have been assigned to a product development project. The performing
organization is well-versed with the traditional waterfall framework but will incorporate scrum for the software development phase of the
project.

What is the best strategy to ensure that all of the scrum roles and responsibilities are served in the project?

Remove the business analyst from the software development phase since that role is only associated with waterfall
project management
Mentor the business analyst to serve in the role of the product owner during the software development phase since those
roles are closely aligned
Request that one of the programmers serve as the voice-of-the-customer for the purposes of prioritizing the items in the
product backlog
Ask the most experienced tester to approve or reject the deliverables during the sprint review as testers are typically most
familiar with the product

Hint Unmarked

Your answer is incorrect


Explanation:

The roles typically associated with scrum include the scrum master, development team, and product owner. In this scenario, the
product owner is the role that is vacant. Although the business analyst is a role that is typically associated with waterfall projects,
the business analyst can also provide support to the team leader and product owner in an agile environment. The business
analyst typically works with the customer to determine and refine project requirements and develop the requirements
documentation. When agile methodologies are employed, the business analyst may also assist with the development and
refinement of the product backlog. With scrum, the product owner is responsible for managing the product backlog and ensuring
that it is prioritized according to business value. In this case, the business analyst will likely have much of the necessary skill set to
serve as the product owner but will be lacking in knowledge and experience with agile. Thus, the scrum master should mentor the
business analyst to close any agile knowledge gaps and ensure that all of the duties related to a product owner are fulfilled.

Question 84 - Qid 6113086, Communications Management, 2. Process, 10.2 Manage Communications

The construction project has many people and specialized teams at the worksite every day. Frequent communication between the team
leads is critical for safety and efficiency.

Which agile ceremony can this project incorporate to increase timely cross-functional communication?

Iteration reviews
Risk-based spikes
Retrospectives
Daily standups

Hint Unmarked

Your answer is incorrect


Explanation:

When traditional, plan-driven projects take on or use agile tools and techniques, the overall project management approach is
considered hybrid. The scenario provides an example of such a project. Certain agile ceremonies can be incorporated into
traditional projects to help the project manager and team in various ways. In the situation described in the question, the project
demands more frequent communication and alignment between the various stakeholders working on the construction site every
day. This communication is essential to ensure there is a coordination of daily activities and a seamless flow of information. Daily
standups are short time-boxed meetings attended by the team and relevant stakeholders. The team members report of work
completed since the last daily standup, work to be done today, and any impediments to progress. This daily communication and
coordination of activities may help the project manager, team, and other individuals and groups working on the construction site
maintain safety and efficiency, as requested by the question.

Question 85 - Qid 6113087, Communications Management, 2. Process, 10.3 Monitor Communications

At times over a project life cycle, the team must adopt a few agile ceremonies. The aim is to communicate quicker in times of tight
deadlines. Unfortunately, team members are not getting the information needed during the ceremonies.

What is the best option to confirm that the information is received and understood?

Validate with the team that the information was captured and facilitate feedback discussions
Spread out the ceremonies over months between each event to ensure comprehension
Mandate that all project team members update the information radiators after each ceremony
Reduce the number of project team members that attend the ceremonies

Hint Unmarked

Your answer is incorrect


Explanation:

Hybrid projects take the best options from the various project management approaches available. For example, the scenario
describes a traditional project, in which the team is requested to adopt agile ceremonies to speed up communication cycles
during tight deadlines. Occasionally, especially as teams are first learning the methods, there may be gaps in communication and
action. Project managers can assist in improving the effectiveness of agile ceremonies by validating with all the participants that
the information shared with them was received and understood. Confirming that the ceremony is meeting its objectives and
facilitating the necessary feedback loops supports the purpose. Those feedback actions may be gradually lessened as the
ceremonies are internalized by the team and relevant stakeholders.

Question 86 - Qid 6113096, Integration Management, 1. People, 4.0 Project Integration Management

A project manager is wrapping up the hardware development phase of a product development project. The project will now transition to
the software development phase.

How might the project manager's role change with respect to integration management?

Act as a servant leader by determining the relative size of the story points of the items in the product backlog
Delegate the control of the detailed project work planning and product delivery to the project team members
Ensure that the project team members each have specific tasks assigned to them and recorded in a Gantt chart
Strengthen the change management controls to ensure that scoop creep is minimized or entirely eliminated

Hint Unmarked

Your answer is incorrect


Explanation:

The question indicates that a hybrid project is moving from a predictive phase to an agile phase. A project manager's role will
change along with the change in the project management approach. The project manager's role with a predictive phase is
typically more directive and controlling, whereas agile leadership requires servant leadership. Agile teams are considered self-
organizing, such that the project manager would assign work to the project team, and the team would then collaborate to achieve
the project goals. Thus, detailed product planning and delivery will be delegated to the project team. The incorrect answer
choices represent actions that would be considered anti-patterns for agile methodologies.

Question 87 - Qid 6113118, Resource Management, 1. People, 9.4 Develop Team

You are conducting a sprint retrospective in accordance with the communications management plan. The development team members
want to try pair programming as they believe it will improve productivity by reducing rework. Even though you are very skeptical, you agree
to implement the practice as you want to empower the team to be a self-managing unit.

What should you do next?

During the upcoming sprint, measure velocity and, if it decreases, discontinue pair programming and reprimand the team
members
Invite the project sponsor and other key stakeholders to demonstrate the increased collaboration between the project
team members
Submit a change request to update the communications management plan with the new agile practice of pair
programming
At the end of the upcoming sprint, revise the burndown chart, evaluate the productivity, and update the lessons learned
register

Hint Unmarked

Your answer is incorrect


Explanation:

The scenario describes a situation where the development team wants to try pair programming as a strategy to improve
productivity. Even though you, as the project manager, are skeptical that the approach will be effective, you should still support
the project team with their efforts to improve productivity. After all, the purpose of a sprint retrospective is for the project team to
inspect and adapt their practices in order to improve performance. Additionally, the team members are the experts with the
development work and are in a better position to determine how they can best work together compared to a project manager.
After the practice has been implemented, it will be incumbent on the project manager to verify any performance improvements
and bring that information to the project team during the next sprint retrospective so the team can further reflect and adjust. A
burndown chart will capture the team's velocity and allow a before and after comparison to determine whether or not pair
programming was successful in improving productivity. Any knowledge gained should then be captured in the lessons learned
register.

Note, the scenario implies the project is managed using hybrid methods. Therefore, incorporating pair programming may or may
not require an approved change request. This would depend on how explicit the project management plan is in this case.

Question 88 - Qid 6113123, Integration Management, 1. People, 4.3 Direct and Manage Project Work

A project manager is leading a daily standup meeting with a scrum team as part of the Direct and Manage Project Work process. A developer
raises an impediment stating that he is struggling with the software code necessary to authenticate users on the sign-in page. The
developer indicates that the impediment could result in a schedule delay to the current sprint.

What is the project manager's best course of action?

Write the software code for the developer since it is the project manager's responsibility to remove impediments
Submit a change request to update the schedule baseline to reflect the delay caused by the impediment
Move the user authentication feature back into the project backlog and ask the product owner to reprioritize it
Recommend that the scrum team members use swarming as a technique to clear the impediment

Hint Unmarked

Your answer is incorrect


Explanation:

The scenario describes a hybrid project that incorporates agile practices into a predictive project management framework. A
developer raises an impediment during a daily standup meeting. Even though this issue represents a blocker for a particular
developer and may result in a schedule delay, it might not be an impediment for the entire project team. Agile teams emphasize
the use of generalizing specialists. Thus, it is possible that other team members might be able to assist in resolving this issue. The
project team is responsible for producing the deliverables, collaborating on achieving the project's objectives. Swarming is a
technique that is typically used when agile practices are incorporated into a project. With this technique, multiple team members
focus collectively on resolving a specific impediment.

Note, when all of the team members leave their current tasks to help one developer, it is reasonable to assume that their tasks
may also be delayed. While this assumption makes the correct answer arguable, of the choices provided, suggesting swarming is
the project manager's best course of action.

Question 89 - Qid 6113125, Scope Management, 1. People, 5.2 Collect Requirements

Collecting requirements for the project and creating the WBS has been a daunting task because stakeholders' needs are vague and are only
expressed in the most general terms due to the nature of the project. As a result of this uncertainty, the project manager is concerned about
excessive changes, wasted work, and rework anticipated during project execution.

What is the project manager's best course of action?

Develop a project scope statement and decompose it down to the work package level as part of initial project planning
Request that the project team use an iterative and incremental approach to product development with short feedback
loops
Implement a robust change control system to only allow for the most important changes to be incorporated into the
product
Generate an activity list from what is currently known and use the Kanban board to break the activities down to their tasks

Hint Unmarked

Your answer is incorrect


Explanation:

The scenario describes a situation where, due to the nature of the project, the requirements are not clearly understood and can
only be expressed at a high level. This situation presents an obstacle to collecting project requirements and creating a fully
decomposed work breakdown structure (WBS) as part of initial project planning. Projects with a high level of uncertainty in
requirements benefit from iterative and incremental approaches that allow the project scope to evolve over the course of the
project. Demonstrating smaller increments of the product to the relevant stakeholders allows for short feedback loops to keep the
evolving product or result of the project in alignment with stakeholder needs and expectations. For example, the project team
might use prototypes to gain early feedback on the requirements. All of the incorrect answer choices represent strategies that
would be unrealistic for a project with a high degree of uncertainty in the project requirements. Note that incorporating an agile
lifecycle within a project management plan would make this a hybrid project.

Question 90 - Qid 6113126, Scope Management, 1. People, 5.2 Collect Requirements

A project manager is implementing the requirements management plan for a software upgrade project that will incorporate some agile
elements. The project manager needs to quickly define cross-functional requirements and reconcile stakeholder differences to build a
consensus among the relevant stakeholders.

What is the project manager's best course of action?

Create a questionnaire and send it out to all of the stakeholders to determine the project requirements
Conduct an in-person interview with each of the relevant stakeholders to determine their needs
Facilitate a requirements workshop with all of the relevant stakeholders to develop user stories
Perform a job shadowing exercise with the intended end-users of the existing software

Hint Unmarked

Your answer is incorrect


Explanation:

The question implies that the project manager is carrying out the Collect Requirements process for a hybrid project. Workshops
can be used to quickly define cross-functional requirements and reconcile stakeholder differences. The interactive nature of a
workshop can be effective in helping the project manager build a consensus among the stakeholders. When agile methodologies
are being incorporated into a project, a requirements gathering workshop may take the form of a story-writing workshop, where
required functionality is captured in the form of a user story. Of the available choices, only a requirements workshop would meet
the project manager's criteria for being able to 'quickly' gather feedback, and have the interactivity necessary to reconcile
stakeholder differences and help build a consensus.

Question 91 - Qid 6113130, Integration Management, 3. Business Environment, 2.4 Organizational Systems

The organization?s leadership wants projects to include more agile methodologies.

Before mandating any agile methods to be used in a project, what should be done first?

Pause current projects to align them to the new agile cadence


Ask employees to vote on who wants to take part in agile projects
Assess organizational culture and readiness for the transformation
Provide agile training to those who will be part of the pilot project team

Hint Unmarked

Your answer is incorrect


Explanation:

Organizations seeking to blend a traditional plan-driven project management approach with certain agile methodologies should
first assess their organizational culture and readiness to undertake such an initiative. Evolving the organization from carrying out
its projects using one project management approach to another takes time and is recommended to undertake gradually and
incrementally. In the scenario, senior leadership within the organization is promoting the inclusion of more agile practices into the
current project methodologies. Such an organizational change to hybrid modalities may or may not be difficult. To know the
extent of the difficulty an organization may endure, assessments of the organization?s readiness for learning, adapting, and
applying new methods must be done first.

Question 92 - Qid 6113133, Risk Management, 2. Process, 11.3 Perform Qualitative Risk Analysis

You are establishing the relative priorities of individual risks identified in the project.

Which of the following tasks is the least likely to impact this prioritization?

Reviewing the assumption log


Examining the risk register
Researching the stakeholder register
Creating the risk report

Hint Unmarked

Your answer is incorrect


Explanation:

Establishing the relative priorities of individual project risks, an activity described in the scenario, is done as part of the Perform
Qualitative Risk Analysis process. This process prioritizes the risks for further analysis or action by assessing their probability of
occurrence and impact as well as other characteristics. The assumption log, risk register, and stakeholder register are all project
documents which may serve as inputs into the Perform Qualitative Risk Analysis process. Note, the question is asking for the
'least' likely activity to be performed as part of this process. The risk report is a project document that provides information on
sources of overall project risk as well as the summary information on individual project risks and is created as an output from the
Identify Risks process, rather than being used as an input to the Perform Qualitative Risk Analysis process, making the risk report
the best answer to the question asked.
Question 93 - Qid 6113135, Integration Management, 3. Business Environment, 1.2 Foundational Elements

Team members are frequently changing during the long timeline of a multi-phase project. The project manager is questioning whether to
include agile practices to accommodate the changes.

What should the project manager do prior to moving to a hybrid approach?

Ensure project finances are secured for the next project phase
Reduce the team size to nine people, plus or minus two
Target a smaller project as a proof of concept to test the feasibility
Evaluate the benefits of incorporating agile practices

Hint Unmarked

Your answer is incorrect


Explanation:

Shifting the management of a project to a hybrid approach is a decision that identifies and evaluates the potential benefits and
value, along with any impediments, detriments, and consequences. Weighing the pros and cons of a hybrid approach enables the
project manager to understand and appreciate the reason for the new approach. This evaluation also ensures other stakeholders
beyond the project team members are on board with the decision and its rationale.

Question 94 - Qid 6113136, Schedule Management, 3. Business Environment, 6.1 Plan Schedule Management

Midway into project execution, new cybersecurity laws take an immediate effect and require audits of the system. The project has a well
thought out project management plan. The audits may come at any time throughout the system?s development.

What is the best way for the project manager and team to address the risk of audits disrupting the project plans?

Handle the audits as an unplanned spike


Reschedule the audits to the end of the project
Request that the sponsor removes the audits
Inform stakeholders that the project will miss its objectives

Hint Unmarked

Your answer is incorrect


Explanation:

Plan-driven managed projects can utilize some of the tools and techniques from agile methodology to support their project
management. Incorporating agile methods with traditional plan-driven methods would make the overall project management
approach hybrid. In the scenario described, the project team could use the agile technique of a spike. A spike is a short timeboxed
effort established to address a risk, explore possible solutions, answer a question, or gather information, to name but a few
common examples. While this may sound counter-intuitive, of the choices provided, addressing the audit as a spike is the best
answer to the question asked.

Question 95 - Qid 6113138, Integration Management, 2. Process, 4.7 Close Project or Phase

A plan-driven project has many deliverables planned to be released at the end of the phase. The project manager wants to ensure that the
deliverables are completed as expected.

Which agile practices can the project manager recommend to the team to make certain that the deliverables are completed and delivered
as expected?

Definition of ready and iteration reviews


User stories and confidence votes
Definition of done and demonstrations
Backlog refinement and retrospectives
Hint Unmarked

Your answer is incorrect


Explanation:

To properly close a project phase or project, appropriate stakeholders sign off that work has been delivered. Techniques in agile
project management methodologies such as "definition of done" and "demonstrations" can also be applied in a plan-driven
project. This approach helps ensure that stakeholders agree at the start of the project that the work is not only completed, but a
shared understanding of done and the criteria to accept the work are exactly defined. Demonstrations serve as checkpoints
throughout the project to demonstrate product increments to the relevant stakeholders, get feedback, and adjust as needed. By
utilizing those two agile practices, project teams can better determine whether or not the project or phase can be successfully
closed.

Question 96 - Qid 6113139, Resource Management, 2. Process, 9.5 Manage Team

A project manager facilitates a discussion with the team about experimenting with some agile practices in their plan-driven project. A
suggestion of incorporating daily standups is on the table. The team members, however, are unsure of what is expected of them.

What can the project manager suggest the project team members do prior to the daily standup?

Make sure their work progress is reflected in the burndown chart


Submit the necessary progress reports to the product owner
Estimate work items that will be developed after the daily standup
Resolve any defects that have been found in the previous day

Hint Unmarked

Your answer is incorrect


Explanation:

Burndown charts are powerful information radiators used to reflect the team?s progress towards their goals and objectives. In
hybrid managed projects, like the one described in the scenario, the team members can use the burndown charts to track their
progress. To provide an accurate picture of the team?s progress towards the iteration?s goal (assuming the work is done in
iterations), the burndown chart should be updated regularly. Ideally, the burndown chart reflects the team?s daily progress
towards their commitments. Updating their progress prior to the daily standup meetings enables the team to collectively see their
progress and inform their decisions as to what needs their attention and what to work on next. Whether using software tools to
automatically update the burndown chart or manually updating the burndown chart, the responsibility within the team is for all
to update in a timely fashion.

Question 97 - Qid 6113142, Risk Management, 2. Process, 11.6 Implement Risk Responses

The project team is undergoing an agile transformation. This requires the team members to maintain their original project plans in the
current management tool, but also enter performance data in the new tool built around agile practices. The team identifies that the
information in the two tools is out of sync. This risk has been identified earlier in project planning.

What is the best course of action for the project manager?

Accept the risk since agile values individuals and interactions over processes and tools
Recognize that the risk has now become an issue and implement the planned response
Conduct a probability and impact assessment to determine a risk score
Communicate to the project sponsor that work may be delayed due to clerical errors

Hint Unmarked

Your answer is incorrect


Explanation:
Uncertainty is part of every project. In this situation, the team had identified that there is a risk of information not being properly
maintained in the duplicate software systems. The identified risk was added to the project?s risk register, as implied from the
scenario. Once the risk is realized, the risk becomes an issue. As an issue, the project team applies the risk response, if one was
planned, or addresses the issue as part of the appropriate issue management process established for the project. In the scenario
described, the team can take action to reconcile the differences in data integrity from the two project management information
systems (PMIS). Once rectified and the resolution accepted, the issue and the risk can be marked as resolved.

Question 98 - Qid 6113143, Integration Management, 2. Process, 2.4 Organizational Systems

Projects in a program are using various project management approaches, such as plan-driven, agile, hybrid. The project teams are
complaining that because of that, the team members must enter the same work, hours, risks, and other data in multiple tools. The
redundant work is impeding on their productivity time.

What is the best course of action for the program manager?

Direct the teams to continue entering the data in the various systems
Determine what tools and data provide the appropriate oversight
Switch all projects in the program to the agile project management approach
Pause all data entry until a valuable solution can be determined

Hint Unmarked

Your answer is incorrect


Explanation:

Data and metrics are very important to track, monitor, and control any project. This is true regardless of which project
management approach is used. When teams and organizations shift from one approach to another or use various approaches for
different projects with a program, it is common for multiple tools and software to be used. The intent of these tools is to assist the
teams, stakeholders, and project organizations in managing their data and methodology. However, too many or divergent tools
may cause more administration and maintenance. Therefore, project and program governance must determine which data is
most relevant and worth the teams' time. The balance between collecting and communicating project data with the resource load
to enter and upkeep the data can be difficult, but needed decision.

Question 99 - Qid 6113144, Integration Management, 2. Process, 1.2 Foundational Elements

An organization is to undertake a multi-phased project. The sponsor wants to pursue the project objectives as they have been planned but
also wants to accommodate and adjust to new information and changes in the environment as the project progresses.

Which project management approach should the project manager select for this project?

Waterfall
Agile
Hybrid
Phased

Hint Unmarked

Your answer is incorrect


Explanation:

Determining the most appropriate methodology to manage a project can be difficult. Fortunately, project managers have plenty
of tools in their toolbox. One option available to project managers is the ability to blend or utilize various aspects, approaches,
and methods from various project management methodologies. This is referred to as a hybrid approach. In the scenario
described, going forward with the upfront-developed plans and be agile to incoming data and changing environmental conditions
is ripe for a hybrid approach.
Question 100 - Qid 6150814, Risk Management, 3. Business Environment, 11.4 Perform Quantitative Risk Analysis

Your sponsor has committed to building a high-speed rail from one major inland city, Azules, to the coast. You have two possible routes,
either Brazos or Corazon.

Given the costs, odds, and rewards of high demand and low demand of each route in this decision tree, what is the expected monetary
value of the route to Brazos?

1.6 billion
-2 billion
0.8 billion
10.8 billion

Hint Unmarked

Your answer is incorrect


Explanation:

The expected monetary value is an important calculation in performing decision tree analysis as part of the Perform Quantitative
Risk Analysis process. You must account for not only the cost of the decision but the probability and payoff for each path.

In a decision tree, the EMV for a decision is calculated by multiplying the value of each possible profit outcome by its probability of
occurrence and adding the products together.

The value of each possible outcome is the net path value. Remember: when you "net" something, you should consider both
positive and negative flows. The formula to compute net path value is to subtract the path reward minus the cost of its decision.

For building the route to Brazos, here are the steps to compute the expected monetary value:

EMV = 0.8(12B - 10B) + 0.2(6B - 10B)

EMV = 0.8(2B) + 0.2(-4B)

EMV = 1.6B - 0.8B

EMV = 0.8B

Other answer choices do not properly factor the net path value (the payoff minus the cost of the decision) and sum the effects.

Below is the completed decision tree with all the net path values and the resulting expected monetary value of each choice. The
EMV for the Brazos route is 0.8 billion, and the EMV for the Corazon route is -8.8 billion (note this is a negative number). Since the
path with the largest expected monetary value is the Brazos route at 0.8 billion, that is the best choice.
Question 101 - Qid 6113147, Integration Management, 2. Process, 4.4 Manage Project Knowledge

Senior leadership has returned from a conference where they have learned about the agile practice of retrospectives. Even though the
organization does not manage any projects in an agile manner, they mandate that from now on this ceremony is included in all projects.

How should the outputs from the retrospectives be stored and managed?

Senior leadership determines the location of the outputs and who may access the contents
The project team stores the outputs in a location that the project manager can control and limit changes
The project team determines a location that is accessible for relevant stakeholders to view and track
The project manager establishes the location and allows the customer to view and make recommendations

Hint Unmarked

Your answer is incorrect


Explanation:

Retrospectives are useful ceremonies ? whether in an agile project or a hybrid project such as the one(s) described in the scenario.
Retrospectives can be conducted at a regular cadence throughout a project (regardless of whether the deliverables are produced
using iterations or not) to gather lessons learned and identify ways for the project team to adapt and improve. The aims of
retrospectives are to find ways for the team to acknowledge what they are doing well and improve where they can. The outputs
generated from the findings are meant mainly for the project team to reflect on and take necessary actions. Visibility and easy
accessibility is key to ensuring that the outputs are not discarded or forgotten but actioned upon.

Question 102 - Qid 6113155, Resource Management, 1. People, 9.0 Project Resource Management

The resource management plan for an international project calls for a scrum team to be colocated during the regular workweek. However,
after five sprints were completed, the project manager got a change request approved to cut costs by transitioning the team to a virtual
environment working remotely from their homes with the start of the sixth sprint.

After the team begins working remotely, what will be the best method to understand the impact of the change on the team's productivity?

Check the product burndown chart to look for any changes to the trendline
Calculate the CPI at the end of each sprint to determine the change in the team's productivity
Review the product burnup chart to compare velocity before and after the change
Update the project schedule network diagram to reflect the changes in the team's environment

Hint Unmarked

Your answer is incorrect


Explanation:

The scenario describes a hybrid project where a change in the project team environment might have an impact on the
productivity of the team. Burn charts are one way of visualizing the project's performance over time. Depending on the
information that needs to be understood, different approaches can be used to build burn charts. Sprint burn charts typically show
the story points completed by day over the course of a single iteration. Release burn charts document the work completed for
each iteration over the course of a release. Finally, product burn charts typically display the story points completed for each
iteration over the course of the entire project. Burndown charts record the work remaining (usually in the form of story points)
over time. Burnup charts show the work completed by the project team over time. Burnup charts are better for understanding
team productivity than burndown charts as they document the work completed by the project team. Burndown charts can
provide a distorted view of team productivity as an increase/decrease in project scope will affect the trendline independent of the
work actually accomplished. Therefore, of the available choices, the best way to understand the productivity impact of the
change to the work environment is to review the product burnup chart.

Question 103 - Qid 6113157, Resource Management, 1. People, 9.1 Plan Resource Management

As part of the Plan Resource Management process, project team members hold a brainstorming session to charter working agreements. The
team identifies 37 agreements that include communication guidelines, decision-making and conflict resolution processes, core hours, a
definition of a story point, how and when user stories will move through the Kanban board, etc.

What should the project manager do next?

Submit a change request to revise the resource management plan to incorporate the team's working agreements
Verify that all of the project team members support the working agreements and document the agreements in the project
charter
Review the team's working agreements at the start of every daily standup and during the sprint review meetings
Ask the project team members for their top five to ten agreements and document the information in the team charter

Hint Unmarked

Your answer is incorrect


Explanation:

The question describes a project team that creates the team's working agreements as part of the Plan Resource Management
process. The working agreements represent the team's social contract and should be documented in the form of a team charter.
The team charter is an output of the Plan Resource Management process. One best practice for the team charter is to limit the
working agreements to the top five to ten items. A comprehensive list of 37 agreements will likely be too cumbersome to be
effective, and the really important agreements may get lost among less important items. Therefore, it would be appropriate to ask
the project team for their top five to ten agreements and then capture them in the form of a team charter.

Note, the scenario presents a hybrid project in which elements of the traditional project management intertwine with agile
practices. While such a combination may confuse, this information is not required to answer the question correctly. The
information is provided as an example to demonstrate the various project management approaches.

Question 104 - Qid 6113159, Resource Management, 1. People, 9.5 Manage Team

A project manager implements the project management plan meticulously but is struggling with leading the project team effectively. There
is a great deal of unhealthy tension between the project team members as well as with the project manager. The project manager has had
several outbursts during sprint retrospectives, which has caused the meeting to be abruptly adjourned.

What should the project manager do first?

Submit a change request to revise the project management plan to eliminate the requirement for retrospectives
Take a proactive step in improving his own emotional intelligence (EI) to become a more effective leader
Reduce the story size by splitting user stories and use relative estimation with the entire team to estimate
Lead a training session with the project team members to help them become emotionally competent

Hint Unmarked

Your answer is incorrect


Explanation:

The scenario describes a hybrid project carried out according to the project management plan with the deliverables developed
using sprints. The question implies that the project manager lacks emotional competence, as exemplified by his failure to control
his emotions during the sprint retrospectives. The project manager must learn to control his own emotions and lead by example
before he can expect emotional competence from the project team members. Taking action, such as investing in training to
improve his emotional intelligence (EI), would be one positive step in addressing the leadership problem. Once the project
manager has addressed his own lack of emotional competence, he can then work with the project team members to develop the
team's EI. Improving EI will likely have a positive effect on the turnover rate as well.

Question 105 - Qid 6113216, Stakeholder Management, 2. Process, 13.2 Plan Stakeholder Engagement

A project is six months into its nine-month projected duration. The project manager reports that the project is at least three months behind
schedule because certain key stakeholders were resistant to consequential changes due to the project.

What might have been done differently during project planning to avoid this situation?

Better execution of the Manage Stakeholder Engagement process


Development of a stakeholder engagement assessment matrix
Creation of a comprehensive stakeholder register
Establishment of a more robust communications management plan

Hint Unmarked

Your answer is incorrect


Explanation:

The scenario describes a situation where the resistance of certain key stakeholders has caused the project to run behind schedule
and is asking what might have been done differently during project planning. Two of the answer choices represent activities,
which are not associated with the Planning Process Group, and, as a result, those responses can be eliminated. Both of the
remaining answer choices represent plausible actions that may have led to better stakeholder engagement. The actual PMP exam
may present questions with more than one correct answer, and the PMP aspirant must select the 'best' answer among the
potentially correct options. In this case, the development of the stakeholder engagement assessment matrix directly addresses
the issue described by the question.

The stakeholder engagement assessment matrix is used as a tool during the Plan Stakeholder Engagement process and identifies
key stakeholders along with their current and desired engagement level. Classifications for stakeholder engagement may include
unaware, resistant, neutral, supportive, and leading. In this scenario, certain key stakeholders were resistant to the project, which
caused the project to run behind schedule. A stakeholder engagement assessment matrix is the best tool for identifying resistant
stakeholders with enough influence to undermine a project. In this case, identifying the current and desired engagement levels of
the key stakeholders might have allowed the project manager to take action to address the undesirable engagement levels and
prevent the stakeholder's resistance from negatively impact the project's performance. Therefore, among the available options,
the development of a stakeholder engagement assessment matrix is the best answer to the question asked.

Question 106 - Qid 6113623, Schedule Management, 2. Process, 6.3 Sequence Activities

Two team members are arguing over how to organize the work packages in the work breakdown structure (WBS). One says the work
packages should be arranged in the order the work is planned to be carried out, while the other insists the order of the work packages in the
WBS does not matter. The two ask the project manager for advice.

What is the project manager's best response?

Work is decomposed and sequenced in the Define Activities process.


Any approach is good as long as the team members avoid the conflict.
The chronological order of tasks is defined in the Sequence Activities process.
The approach should be defined by the project management office (PMO).

Hint Unmarked

Your answer is incorrect


Explanation:

To answer this question correctly, one should be able to distinguish a deliverable and work package from the schedule and
activities. The planned work is contained within the lowest level of the components of the work breakdown structure (WBS).
These components are called work packages. A work package can be used to group the activities where work is scheduled and
estimated, monitored, and controlled. In the context of the WBS, work refers to work products or deliverables that are the result of
activity and not to the activity itself. The work packages are further decomposed into schedule activities that are then sequenced
to obtain the greatest efficiency given all the project's constraints. The Create WBS process provides a framework of 'what' has to
be delivered but does not involve sequencing.

Question 107 - Qid 6113718, Resource Management, 2. Process, 9.3 Acquire Resources

You are leading a project to design a high-efficiency solar cell. The project needs copper, silver, and silicon among other materials. You have
ordered custom-built manufacturing equipment, secured permission to install a solar array on the roof for testing, and reserved a
conference room for daily stand-up meetings.

What is the result of your efforts?

Resource histogram
Resource management plan
Resource assignment matrix
Physical resource assignments

Hint Unmarked

Your answer is incorrect


Explanation:

It can be inferred from the question that the process of acquiring resources is being performed. One of the outputs from the
Acquire Resources process is physical resource assignments. Documentation of the physical resource assignments records the
material, equipment, supplies, locations, and other physical resources that will be used during the project. Based on the
description provided by the question, the physical resource assignments is what has been accomplished.

Question 108 - Qid 6113733, Cost Management, 2. Process, 7.3 Determine Budget

During project execution, several unexpected issues have developed and are now threatening the project schedule and budget.

What is the best course of action for the project manager to address the issues?

Use contingency reserves


Review project funding requirements
Use management reserves
Review organizational process assets

Hint Unmarked

Your answer is incorrect


Explanation:

The total project budget includes the cost baseline plus the management reserves. The cost baseline is comprised of the work
package estimates and contingency reserves. The contingency reserves are allocated for identified risks. Management reserves
are withheld for unforeseen risks that affect the project. According to the scenario, the issues developed during the project were
unexpected. Therefore, of the choices provided, using the management reserves is the best course of action for the project
manager.

Note, the scenario describes issues, while the explanation refers to risks. Prospective PMP aspirants should keep in mind that the
questions on the real PMP exam may have such discrepancy in their wording.

Question 109 - Qid 6113755, Risk Management, 2. Process, 11.2 Identify Risks

To produce the deliverables, the project will procure goods and services from outside the organization. The project manager has instructed
the team to develop a comprehensive list of individual project risks related to those procurements.

What is the best course of action for the project team?


Consult the project charter
Review procurement documentation
Develop a procurement register
Check the probability and impact matrix

Hint Unmarked

Your answer is incorrect


Explanation:

This question involves two Project Management Knowledge Areas: Project Risk Management and Project Procurement
Management. Based on the scenario described, the project team is carrying out the Identify Risks process. Procurement
documentation is an output from the Plan Procurement Management process and an input to the Identify Risks process.
Procurement documentation may include bid documents, the procurement statement of work, payment information, contractor
work performance information, plans, drawings, correspondence. Since procuring goods and services from outside the
organization may increase or decrease overall project risk and may introduce additional individual project risks, reviewing
procurement documentation is the best course of action for the project team.

Question 110 - Qid 6113796, Cost Management, 2. Process, 7.4 Control Costs

You are managing a renovation project of a house which is to be completed at a budget of $100,000. Earned value analysis shows that so far
you have completed 40% of the work and spent $60,000.

How much more will it cost to complete the project if the cost performance remains unchanged?

$160,000
$149,254
$120,000
$89,254

Hint Unmarked

Your answer is incorrect


Explanation:

The total budget, or budget at completion (BAC), is $100,000. The cost so far, or actual cost (AC), is $60,000. You have completed
40% of the work which is the earned value (EV), i.e., EV = 40% x $100,000 = $40,000.

Understanding what exactly the question asks is the key to answering the question correctly. The last sentence of the question
starts with "How much more will it cost to complete the project...", implying we are requested to find the estimate to complete
(ETC). The ETC can be calculated as follows:

ETC = EAC - AC, where the EAC is the estimate to complete.

Selecting the correct formual for the EAC is another key to answering this question correctly. The last sentence ofthe question
ends with "...if the cost performance remains unchanged?", implying that the formula for the EAC should consider the cost
performance index (CPI), i.e.:

EAC = BAC / CPI

The CPI, in turn, can be claculated using the following formula:

CPI = EV / AC = 40,000 / 60,000 = 0.67

The rest of the calculations are shown below:

EAC = 100,000 / 0.67 = 149,254

ETC = 149,254 - 60,000 = 89,254

Therefore, the cost to complete the renovation if the cost performance remains unchanged is $89,254
Question 111 - Qid 6125212, Procurement Management, 2. Process, 12.3 Control Procurements

A major dispute arises with a vendor on a project. The project manager claims that the vendor has failed to perform its contractual
obligations, while the vendor complains that the project is behind on payments.

What is the best course of action for the project manager in this situation?

Refer this issue to a third party for mediation or arbitration


File a contract claim and then work directly with the vendor to resolve the issue
Terminate the contract, document lessons learned, and find another vendor
Conduct a procurement audit

Hint Unmarked

Your answer is incorrect


Explanation:

When disputes arise on a project, claims administration can be an effective tool to help buyers and sellers reach an agreement
and prevent damage caused by litigation and early contract termination. Filing a contract claim and working with the vendor to
reach an agreement is a more direct problem-solving approach and therefore the 'best' answer choice. Claims administration also
refers to using alternative dispute resolution. However, since mediation and arbitration can be costly, risky, and time-consuming,
of the choices provided, negotiation is the preferred method of settling disputes and is, therefore, the best course of action in this
situation.

Question 112 - Qid 6125216, Resource Management, 1. People, 9.5 Manage Team

You want to recognize the contributions of some of the high-performing team members so far in the project.

Which of the following do you need to guide you through this process?

Organizational process assets


Stakeholder register
Project team assignments
Quality metrics

Hint Unmarked

Your answer is incorrect


Explanation:

The scenario implies you are carrying out the Manage Team process. Manage Team is the process of tracking team member
performance, providing feedback, resolving issues, and managing team changes. Organizational process assets (OPAs) are an
input to this process. Examples of the OPAs associated with the Manage Team process include certificates of appreciation,
corporate apparel, and other organizational perquisites. Recognizing the contributions of the high-performing team members is
part of the Manage Team process. The project manager can use the relevant OPAs to guide her through this process and provide
the team members with the appropriate rewards and recognition.

Question 113 - Qid 6150715, Schedule Management, 2. Process, 6.2 Define Activities

You are managing a project to construct a bridge at a location that is highly regulated by multiple levels of government. You are in the
process of developing a plan that will establish the criteria and the activities for developing, monitoring, and controlling the schedule.

Which of the following should you gather to complete this process?

The charter, project management plan, estimation databases, and PMO templates
Government standards, the assumption log, schedule management plan, and milestone list
The resource breakdown structure, lessons learned register, and scheduling methodology
The project scope statement, risk register, and project team assignments

Hint Unmarked
Your answer is incorrect
Explanation:

Establishing the criteria and the activities for developing, monitoring, and controlling the schedule is the main purpose of
developing the schedule management plan, which is the primary output of the Plan Schedule Management process. To develop
this plan, four inputs are required: project charter, project management plan, enterprise environmental factors (EEFs), and
organizational process assets (OPAs). The project charter and the project management plan are explicitly called out in the correct
answer choice, whereas the EEFs and the OPAs are represented by the estimation databases and PMO templates correspondingly.
Therefore, among the available options, the charter, the project management plan, estimation databases, and PMO templates
represent the best answer to the question asked.

Question 114 - Qid 6151376, Resource Management, 2. Process, 9.2 Estimate Activity Resources

One of the project activities requires that a search of a large area is conducted to find debris from an explosion of a failed prototype. The
project is now in planning, and there is a debate among the team members on the type and quantity of resources needed to complete this
activity.

What should the project manager do first?

Select any resource and quantity to complete the task by the due date
Request the team to vote and make a decision based on majority
Research various levels of resource capability and skills, types, and sizes
Utilize a PMIS to manage resource pools and allocate resources

Hint Unmarked

Your answer is incorrect


Explanation:

The question implies the team is carrying out the Estimate Activity Resources process. This process uses many tools and
techniques. One technique to understand the resource needs and the best potential solutions for the activity resources is data
analysis. Data analysis can be conducted in many forms and formats. The objective is to evaluate possible options or approaches
or values to determine the best activity resource estimates. Once a value or a range of values has been determined as reasonable,
estimates can be set or decisions that are most suitable for the participants made. Therefore, of the choices provided, before
making any decision, researching various resources levels, types, and methods is what the project manager should do first.

Question 115 - Qid 6151382, Cost Management, 2. Process, 7.1 Plan Cost Management

A project manager is developing a cost management plan and needs to determine the best source of funding for a project that is dictated by
a legal requirement. The cost of capital is estimated at 9.7% for non-dividend paying equity, 6.7% for debt, and 5.1% for self-funding. The
NPV of the project is $500,000, and the opportunity cost is $750,000.

What is the project manager's best course of action?

Fund the project with equity since there are no dividend obligations
Select the self-funding option since it provides the lowest cost of capital
Perform an alternatives analysis since there are multiple factors to consider
Recommend the termination of the project since another project has a higher NPV

Hint Unmarked

Your answer is incorrect


Explanation:

The question states that the project manager is developing the cost management plan, which indicates that the Plan Cost
Management process is being performed. A data analysis technique that can be used for this process includes an alternatives
analysis, which is a technique used to evaluate identified options in order to select the options or approaches to use to execute
and perform the work of the project. In this case, an alternatives analysis can consist of reviewing funding options such as self-
funding, funding with equity, or funding with debt. Even though the question provides some financial data, there are other factors
which should be taken into consideration before selecting the appropriate source of funding. Therefore, among the available
options, the best course of action for the project manager is to perform an alternatives analysis.

Question 116 - Qid 6151383, Cost Management, 3. Business Environment, 7.2 Estimate Costs

You are in the process of developing an approximation of the monetary resources needed to complete project work for a large-scale
multinational project which will take at least seven years to complete. Your previous projects have all been domestic with short timeframes.

As part of the process you are currently performing, what might you need to do differently compared with your past projects?

Create a stakeholder engagement assessment matrix


Develop a more robust risk management plan
Consider additional enterprise environmental factors
Include additional organizational process assets

Hint Unmarked

Your answer is incorrect


Explanation:

Developing an approximation of the monetary resources needed to complete project work is the main objective of the Estimate
Costs process. Therefore, the question implies that you are conducting the Estimate Costs process and asks how this process will
be affected by a large long-term multinational project. Thus, the answer choices that relate to other project management
processes can be eliminated as correct responses. With the project lasting at least seven years, the impact of inflation over the
course of the project must be considered. Additionally, since the project is international in scope, the project will likely be working
with various currencies that will be subject to fluctuations in relative values. For large-scale projects that extend multiple years
with multiple currencies, the fluctuations of currencies and inflation need to be understood and built into the Estimate Costs
process. Exchange rates and inflation are considered enterprise environmental factors. Therefore, of the available choices,
considering additional enterprise environmental factors is the best response.

Question 117 - Qid 6151389, Scope Management, 2. Process, 5.5 Validate Scope

You are assigned to a software development project, and you are conducting a product review with an important client. Although the
deliverables were previously verified, the client detects a defect and rejects one of the deliverables.

What should you do?

Submit a change request to fix the deliverable


Refund the customer for the value of the deliverable
Ask the project team to repair the defect
Begin the claims administration process

Hint Unmarked

Your answer is incorrect


Explanation:

The question states that the deliverables have been previously verified implying the Control Quality process has been completed
and resulted in verified deliverables as one of its outputs. In the scenario described, you are performing the Validate Scope
process which is the process of formalizing acceptance of completed and verified project deliverables. One of the tools and
techniques associated with this process is an inspection. The inspection includes activities such as measuring, examining, and
validating to determine whether the work and deliverables meet requirements and product acceptance criteria. A product review
is an example of an inspection. In this scenario, the client performed a test which exposed a software bug and caused a
deliverable to be rejected. Unless stated otherwise in the question text, deliverables are always placed under configuration
control. Therefore, submitting a change request to fix the deliverable is what you should do.
Question 118 - Qid 6151393, Integration Management, 1. People, 4.3 Direct and Manage Project Work

Not all team members are properly checking out resources used in the project.

What is the best option for the project manager?

Wait until there is a need or the team is impacted


Escalate the issue to the functional managers
Discuss the checkout rationale with the team
Remove the process of checking out resources

Hint Unmarked

Your answer is incorrect


Explanation:

There is not enough information in the question to determine what resources exactly the scenario is talking about, and what does
this checkout procedure mean. Prospective PMP aspirants have to keep in mind that questions on the real PMP exam may be
short and vague. This question is a good example of what you may encounter on your exam. Regardless of how ambiguous the
scenario is, always try to select the best answer to the question asked even if none of the choices looks like a perfect/ideal answer.

During the Direct and Manage Project Work process, the project manager is guiding, directing, and facilitating the plans of the
project. Based on the scenario, it appears that the plans require the resources to be checked out. Those plans need to be better
communicated and understood. There are many actions the project manager could take to direct and manage the project team to
follow through with those plans. The answer choice that best aligns to that need is to discuss the purpose and rationale for the
resource checkout and why the team should follow those plans.

Question 119 - Qid 6151395, Scope Management, 2. Process, 5.6 Control Scope

In a pharmaceutical project, human trials have been started. The CEO is thrilled about the success of the trials up to this point. She
announces that the size of the patient pool for the trials should double from the one originally planned and results tracked in the new
system launching this week.

What should the project manager do first?

Ramp up production to match the demand of the CEO


Consider what might be included in a change request
Adjust the scope baseline to reflect the increased trial size
Communicate the change in the scope to the project stakeholders

Hint Unmarked

Your answer is incorrect


Explanation:

Managing changes to the project by following the organization's change control process established for the project is important in
keeping the project under control and operating effectively. Changes to scope, like in the scenario of doubling the size of the
patient pool as well as tracking the results in the new system, require evaluation, agreement, and approval. Who makes those
approvals is up to the project organization and its change management plans. But before any changes are implemented or
communicated, the change request must be submitted per the project's procedure and approved by those authorized to do so. A
CEO's declaration of what should happen in the project does not substitute for the project's change control procedures and the
Perform Integrated Change Control process.

Question 120 - Qid 6114007, Schedule Management, 2. Process, 6.5 Develop Schedule

Following is an illustration of a project network activity node that includes some information about activity D.

Given the information provided, what is the total float of activity D?


2
14
16
18

Hint Marked

Your answer is incorrect


Explanation:

To answer this question, the first step is to understand the meaning of the values displayed on the network activity node. The
accepted convention is shown below, where:

ES = early start
EF = early finish
LS = late start
LF = late finish

The total float can be calculated using either one of the following two formulas:

Total Float = LF - EF
Total Float = LS - ES

To illustrate, we will use the first formula:

Total Float = LF - EF = 39 - 23 = 16

Here is the project schedule network activity node filled in with the correct answer for total float, 16:
Please note that on this page we only show a general explanation for the 120 questions.

The PMP Exam Simulator (http://www.pm-exam-simulator.com/pmp), however, will give you a much richer experience
because you also receive an explanation of why each of the answer choices is correct or incorrect, as well as a reference that
allows you to read up on the subject and learn from any mistakes.

Try The PM Exam Simulator


Do you want to prepare for your PMP exam in the best possible way? Then why not consider The PM Exam Simulator. It offers 2,100 high-
quality questions in the most modern and realistic online environment. Thousands of students have passed their exam after using this tool:

(/component/banners/click/19)

Get The PMP Exam Simulator (/component/banners/click/19)


 Instructions
Test your exam-readiness with these 120 free PMP questions:

You have 1.2 mins per question


Select the best answer per question
Mark questions for review
Use "hint" if you are stuck
Unanswered questions count as incorrect
Perfect for PMP Practice Exam (/component/banners/click/17) preparation

Click the red button at the bottom of the page to end the exam and see your results.

 Restart the Exam

Clear All & Restart Exam! (https://www.project-management-prepcast.com/pmp-practice-exam-questions-sample-test/)

Your Result :
4%
You answered 5 out of 120 questions correctly.
You have 115 incorrect, 113 of which were unanswered.
7 questions marked

Want More?
The PM Exam Simulator offers 2,280 questions in a realistic, online environment. Try it!

Get PM Exam Simulator (/component/banners/click/20)


OSP INTERNATIONAL LLC
Training for Project Management Professional (PMP)®, PMI Agile Certified Practitioner (PMI-ACP)®, and Certified Associate in Project Management (CAPM)®

 (https://twitter.com/pmexamtips)  (https://www.facebook.com/PrepCast)
 (https://www.pinterest.com/pmexamtips/)
 (https://www.youtube.com/user/CorneliusFichtner)

Company Info Customer Service


About Us (/about-us) Contact Us (/contact)

Our Team (/team) Help (https://pmhelpdesk.zendesk.com/hc/en-us)

Our Mission (/our-mission) Search (/search)

Terms & Conditions (http://www.osp-international.com/tc) Sitemap (/sitemap)

Privacy Notice (http://www.osp-international.com/privacynotice) Corporate Project Management Training (/project-management-corporate-


training)

Group Discounts (/group-discounts)

PM PrepCast Discount Coupon (/discount-coupon)

Scholarships (/scholarships)

Resources Exam Prep Training


Free PMP® Newsletter (/free/pmp-exam-newsletter) PMP® Exam Simulator (https://www.pm-exam-simulator.com)

Free PMP® Questions (/free/pmp-exam/free-pmp-exam-questions) PMP® Formulas (https://www.project-management-formulas.com)

Free PMI-ACP® Questions (/free/pmi-acp-exam/free-questions) PMI-ACP® Exam Training (http://www.agileprepcast.com/)

Free CAPM® Simulator (/capm) CAPM® Exam Training (/capm)

PMP® Exam Prep (/)

PrepCast PMP® Practice Exam (https://www.pm-exam-simulator.com)

Copyright © 2008 - 2023 OSP International LLC.


PMI, the PMI Authorized Training Partner logo, PMBOK, PMP, PgMP, PfMP, CAPM, PMI-SP, PMI-RMP, PMI-ACP, and PMI-PBA are registered marks of the Project Management Institute, Inc.
Please note that a passing score in an Exam or Quiz of The PM Exam Simulator™ is no guarantee that you will pass your PMI Exam
 

You might also like